PATHO MID-TERM

Ace your homework & exams now with Quizwiz!

Which student statement demonstrates a sound understanding of the cellular processes of hypertrophy and hyperplasia? - "A remaining kidney can sometimes undergo hyperplasia in response to one that has been removed" - "Clients with cardiomyopathy undergo myocardial hypertrophy with proportional increases in cell length and width" - "When male clients experience an enlarge prostate, they are describing a form of hypertrophy" - "I know that cells like neurons have little capacity for hyperplastic growth" chapter 3

"I know that cells like neurons have little capacity for hyperplastic growth"

As part of a first aid class, a health care instructor is teaching a group of industrial workers about how electrical injuries can cause cell damage. Which of the statements made by one of the workers indicate that further teaching is necessary? - "the most severe damage is likely to occur where the current enters and leaves the body: - "the greater the skin resistance, the greater the amount of deep and systemic damage a victim is likely to incur" - "resistance to flow is the phenomenon that transform electrical energy into heat - "the particular pathway that a current takes through the body is very significant" chapter 3

"the greater the skin resistance, the greater the amount of deep and systemic damage a victim is likely to incur"

Which clients would be at risk for developing nonthrombocytopenic purpura? Select all that apply. - 55-year-old client diagnosed with Cushing disease displaying bruises, weight gain with buffalo hump, and "moon" face - 73-year-old client admitted with concussion that resulted from a fall - Pregnant mother experiencing headaches and proteinuria - 15-year-old with insulin-dependent diabetes who has hyperglycemia and is displaying irritability with headaches and tachycardia - Child adopted from India and displaying malaise, lethargy , and petechiae all over the body associated with suspected scurvy

- 55-year-old client diagnosed with Cushing disease displaying bruises, weight gain with buffalo hump, and "moon" face - Child adopted from India and displaying malaise, lethargy , and petechiae all over the body associated with suspected scurvy

Which client may be experiencing a sensory focal seizure that has sent an abnormal cortical discharge to the autonomic nervous system (ANS)? - 22-year-old reporting a stiff neck and achiness, along with some nausea and vomiting - 56-year-old reporting tingle sensations and has both an elevated pulse and BP - 85-year-old client experiencing drooping of the right side of face and numbness in right arm and leg - 44-year-old client reporting constant movement and pain in the legs that gets worse when attempting to sleep chapter 16

- 56-year-old reporting tingle sensations and has both an elevated pulse and BP

A nurse is developing a program to help clients reduce the impact of chronic stress on their health. Which clients should the nurse prioritize as being able to most benefit from the program? Select all that apply. - A 30-year-old with substance abuse - A 15-year-old with mononucleosis - A 45-year-old with inflammatory disease - A 22-year-old with an eating disorder - A 75-year-old with pneumonia chapter 7

- A 22-year-old with an eating disorder - A 45-year-old with inflammatory disease - A 30-year-old with substance abuse

The nurse is planning care for a group of clients. Which client should the nurse assess first? - A 65-year-old client with a while blood cell count of 15,200 uL - A 25-year-old client with an absolute neutrophil count of 300/mm^3 - An 82 year-old client with a hemoglobin of 9.0g/dL - A 45-year-old client with a platelet count of 100,000/mm^3 chapter 24

- A 25-year-old client with an absolute neutrophil count of 300/mm^3

The nurse is planning care for a group of clients. Which clients should the nurse prioritize as "at risk" for hypercalcemia and advocate for monitoring calcium levels? Select all that apply. - A client with heart failure who is taking a loop diuretic - A client with hypophosphatemia - A client who has breast cancer with bone metastasis - A client with hypoparathyrodism - A client with prolonged immobility chapter 3

- A client with hypophosphatemia - A client who has breast cancer with bone metastasis - A client with prolonged immobility

A parent brings a 12-year-old to the emergency room with multiple bite wounds to the arms and hands from a stray cat. Which interventions should the nurse be prepared to apply? Select all that apply. - Administration of corticosteroids - A review of the client's immunization record - Irrigation of the wounds - Suturing of the wounds - Administration of prophylactic antibiotics - Rabies prophylaxis chapter 9

- A review of the client's immunization record - Irrigation of the wounds - Administration of prophylactic antibiotics - Rabies prophylaxis

An 80-year-old client has a stage 3 decubitus ulcer on the first ischial tuberosity which has not shown much improvement despite optimal local wound treatment. What other interventions should the nurse recommend to promote wound healing? Select all that apply. - Positioning the client in a chair three times a day - Administering corticosteroids - A review of the client's medications - Placing the client on an alternating pressure mattress - Nutritional supplements chapter 9

- A review of the client's medications - Placing the client on an alternating pressure mattress - Nutritional supplements

The nurse is reviewing the treatment plan with parents of a 6-month-old diagnosed with severe B-thalessemia. Which statements by the parents demonstrate they understand the treatment plan? Select all that apply. - We will need to feed our child a diet high in iron to promote red blood cell production - Transfusions are needed to prevent skeletal abnormalities from developing - Transfusions will need to begin once our child reaches 2 years of age - Oxygen therapy should be used to prevent organ damage from chronic hypoxia - A stem cell transplant may be used as part of the treatment plan chapter 23

- A stem cell transplant may be used as part of the treatment plan - Transfusions are needed to prevent skeletal abnormalities from developing

A widow, who lost her husband a few weeks ago, is having trouble with insomnia and maintaining their normal sleep pattern. When visiting with their healthcare provider, they suggest a prescription to help regain her normal circadian pattern. This is based on the fact that interruption of sleep-wake cycles can cause which of the following problems? Select all that apply. - Excess of non-rapid eye movement (NREM) sleep which affects the creativity process. - Hallucinations that may result in harm to the individual. - Restless leg syndrome due to inability to find a comfortable sleeping position. - Alterations in their immune function that can result in an infection. - An increase risk in accidents when sleep deprived similar to those under the influence of alcohol. chapter 7

- Alterations in their immune function that can result in an infection. - An increase risk in accidents when sleep deprived similar to those under the influence of alcohol.

A pregnant client is exploring options related to prenatal testing due to a history of inherited disorders. What information should the nurse include when informing the client about various prenatal tests? Select all that apply. - The type of test is used is determined by your preference and level of invasiveness - If genetic disorders are being investigated, chorionic villus sampling will be required - An ultrasound will be required for either amniocentesis or chorionic villus sampling - All of the tests used to examine fetal DNA are invasive and carry associated risks - Percutaneous umbilical cord blood sampling is needed for a truly accurate results chapter 5

- An ultrasound will be required for either amniocentesis or chorionic villus sampling - All of the tests used to examine fetal DNA are invasive and carry associated risks

The nurse notes eosinophilia on the client's laboratory results. The nurse should interview the client about a history of which conditions? Select all that apply. - Asthma - Bacterial infection - Viral infection - Allergic reactions - Exposure to parasites chapter 21

- Asthma - Allergic reactions - Exposure to parasites

A client is admitted to a rehabilitation center after hospital treatment for an ischemic stroke. What aspects of the client's history are risk factors for ischemic stroke? Select all that apply. - Black male - Diagnosed with type 2 diabetes 8 years ago - Takes iron supplements for the treatment of chronic anemia - Blood pressure historically in the range of 150/90 to 160/100 mm Hg - Takes corticosteroids for the treatment of rheumatoid arthritis chapter 16

- Black male - Diagnosed with type 2 diabetes 8 years ago - Blood pressure historically in the range of 150/90 to 160/100 mm Hg

The nurse is caring for a client who has a tumor that results in excess levels of catecholamines being released. Which assessments will the nurse prioritize in this client's plan of care? Select all that apply. - Blood pressure - Levels of consciousness - Blood clucose - Heart rate and rhythm - Oxygen saturation chapter 7

- Blood pressure - Blood glucose - Heart rate and rhythm

A client arrives at the emergency department with symptoms of stroke. What evidence should the nurse gather to determine if the client is a candidate for thrombolytic therapy? Select all that apply. - Blood pressure - History of stroke - Age - Time of symptom onset - Heart rate chapter 16

- Blood pressure - History of stroke - Time of symptom onset

After years of going to different physicians with vague symptoms, a 55-year-old client with a history of Hodgkin lymphoma has been diagnosed with a secondary immunodeficiency syndrome. The client asks the nurse what this means. The nurse knows that secondary immunodeficiency disorders may occur if which background information is available? Select all that apply - May be inherited as a sex-linked trait - Can result from frequent recurring Staphylococcus aureus infections - Can occur in clients taking corticosteroids daily - Usually develop later in life - May be a result of chemotherapy being used to treat a cancer chapter 12

- Can occur in clients taking corticosteroids daily - Usually develop later in life - May be a result of chemotherapy being used to treat a cancer

Two years after chemotherapy and radiation therapy for lung cancer, a 72-year-old client reports being extremely tired all the time. The physician suspects that the client may have developed aplastic anemia. The nurse assessing the client will likely find which clinical manifestation of aplastic anemia? Select all that apply. - Hemolysis from renal dialysis treatments - Spoon-shaped deformity of the fingernails - Complaints of weakness and fatigue - Excess bleeding from gums and nose - Small spots of skin hemorrhages over entire body chapter 23

- Complaints of weakness and fatigue - Excess bleeding from gums and nose - Small spots of skin hemorrhages over entire body

While traveling throughout Asia, a young couple was exposed to rubella. During their first clinic visit, the couple found out they were pregnant and express concern about their possible exposure to rubella. The nurse knows that this infant is at high risk for which possible complications related to rubella exposure? Select all that apply - Small outbreak of blisters around the eyes and mouth 2 weeks after delivery - Short, flipper-like appendages - Facial deformities like small palpebral fissures or thin vermillion border - Deafness - Blindness or cataracts chapter 5

- Deafness - Blindness or cataracts

While preparing a client about to undergo percutaneous umbilical cord blood sampling (PUBS), which educational information should the nurse provide prior to the procedure? Select all that apply. - We will send a sample of amniotic fluid to a regional medical center to have DNA tests performed for any genetic abnormality - During the procedure, an ultrasound will be utilized to guide the catheter into the correct position - Once the procedure is begun, you must lie very still since they will be inserting a needle through the uterine wall - We will put you into the stirrups and dilate your cervix with a small catheter so that we can obtain a cord sample chapter 5

- During the procedure, an ultrasound will be utilized to guide the catheter into the correct position - Once the procedure is begun, you must lie very still since they will be inserting a needle through the uterine wall

The nurse is assessing a client. What findings should the nurse apply as evidence of iron deficiency anemia? Select all that apply. - Paresthesia of the feet and fingers - Oral lesions - Concave fingernails (koilonychia) - Jaundice - Dyspnea on exertion chapter 23

- Dyspnea on exertion - Oral lesions - Concave fingernails (koilonychia)

The nurse is caring for a teenager with sickle cell anemia. When comparing new laboratory results with the client's baseline, which results should the nurse prioritize as requiring intervention? Select all that apply. - Elevated serum bilirubin - Elevated serum osmolality - Low pH on arterial blood gas - Decreased hemoglobin - Elevated white blood cell count chapter 23

- Elevated serum osmolality - Low pH on arterial blood gas - Elevated white blood cell count

Which assessment findings correlate with the health care providers preliminary diagnosis of a more aggressive form of non-Hodgkin lymphoma (NHL)? Select all that apply. - Enlarge pancreas - Elevated temperature - Peripheral edema - Reed-sternberg cells in lymph node tissue - Night sweats - Weight loss chapter 24

- Elevated temperature - Night sweats - Weight loss

Tumor necrosis factor-a and IL-1 are major cytokines that mediate inflammation. If the client is developing a systemic response to an infection, the nurse will likely assess which clinical manifestations? Select all that apply. - Decrease in urine output - Elevated temperature - Hypertension - Tachycardia - Anorexia chapter 9

- Elevated temperature - Tachycardia - Anorexia

A brain tumor causing clinical manifestations of headache, nausea, projectile vomiting, and mental changes is likely located in which part of the brain? Select all that apply. - Brain stem - Extra-axial - Intra-axial - Temporal lobe - Frontal lobe chapter 16

- Extra-axial - Intra-axial - Frontal lobe

The nurse is planning care for a client in the acute recovery phase after an ischemic stroke. What interventions will the nurse prioritize when planning care? Select all that apply. - Falls prevention - Deep vein prophylaxis - Swallowing precautions - Reorientation exercises - Stroke prevention education chapter 16

- Falls prevention - Deep vein prophylaxis - Swallowing precautions

The nurse is planning care for a client who is receiving antimetabolite chemotherapy. What should the nurse include in the plan of care? Select all that apply. - Encourage intake of a soft diet with fresh fruit and vegetables - Monitor bowel movements closely - Indwelling urinary catheter for accurate output record - Falls prevention protocol - Daily weights chapter 6

- Falls prevention protocol - Daily weights - Monitor bowel movements closely

A school nurse is working with a 16-year-old client recovering from mononucleosis. Which activities should the nurse recommend the teenage avoid while recovering? Select all that apply. - Driving a vehicle - Football - Hockey - Tennis - Swimming chapter 24

- Football - Hockey

From the current health histories, which client(s) is at risk for developing secondary forms of immune thrombocytopenic purpura (ITP)? Select all that apply. - Former IV drug user recently diagnosed with hepatitis C virus - Older adult female undergoing testing for cause of joint discomfort - Middle-aged client scheduled for gastroscopy to confirm Helicobactor pylori - Female client in the first trimester reporting abdominal cramping - Adolescent for facial burns when the "vape" (cigarette) exploded in the mouth chapter 22

- Former IV drug user recently diagnosed with hepatitis C virus - Middle-aged client scheduled for gastroscopy to confirm Helicobactor pylori

The nurse is caring for a client with a leukemia and a blast count of 100,000 cells/uL. What client reports should prompt the nurse to take immediate action? Select all that apply - Increased urine output - Headache - New onset dyspnea - Lethargy - Bleeding from gums chapter 24

- Headache - New onset dyspnea - Lethargy

The nurse is caring for a client with thrombotic thrombocytopenic purpura (TTP) and notices a yellow discoloration of the client's sclera. Which are likely possible causes of this finding? Select all that apply. - Vital hepatitis - Hypoperfusion of the liver - Accelerated intravascular hemolysis - Side effect of hepatotoxic medication - Elevated levels of nitrogenous wastes (uremia) chapter 22

- Hypoperfusion of the liver - Accelerated intravascular hemolysis

While attending an international nursing conference, many discussions and break-out sessions focused on the World Health Organization's (WHO) views on health. Of the following comments made by the nurses during a discussion session, which statements would be considered a good representation of the WHO definition? Select all that apply. - Interests in keeping the older adult population engages in such activities as book reviews and word games during social time - Increase in the number of chair aerobics classes provided in the skilled care facilities - Interventions geared toward keeping the older adult population diagnosed with diabetes mellitus during tight blood glucose control by providing in-home cooking classes - Providing handwashing teaching sessions to a group of young children - Providing transportation for renal dialysis clients to and from their hemodialysis sessions - Chapter 1

- Interests in keeping the older adult population engages in such activities as book reviews and word games during social time - Increase in the number of chair aerobics classes provided in the skilled care facilities - Interventions geared toward keeping the older adult population diagnosed with diabetes mellitus during tight blood glucose control by providing in-home cooking classes - Providing handwashing teaching sessions to a group of young children

A client has started on interferon beta treatment of multiple sclerosis. What should the nurse include in the teaching? Select all that apply. - Interferon beta does not have any known side effects - Interferon beta alters your immune response - Seek emergency treatment if you develop flu-like symptoms - The medication will be given by subcutaneous injection - This drug can reduce symptoms but will not alter disease activity chapter 15

- Interferon beta alters your immune response - The medication will be given by subcutaneous injection

A nurse is providing care for a client who has been diagnosed with metabolic alkalosis after several days of antacid. Which treatment should the nurse prepare to give? - Intravenous or oral administration of free hydrogen ions - Supplementary oxygen and possible mechanical ventilation - Intravenous adminastration of a KCI solution - Administration of oxygen and NaHCO3 solutions chapter 8

- Intravenous adminastration of a KCI solution

The mother of an 18-month-old child is concern that her child is lethargic and not eating foods that he normally enjoys. She takes him to the pediatrician for a check-up. Which clinical manifestations lead the health care provider to suspect the child may have a neuroblastoma? Select all that apply - Excessive burping - Large amount of pale urine - Large, protruding abdomen - Crying when joints/position changed - Weight loss chapter 6

- Large, protruding abdomen - Crying when joints/position changed - Weight loss

A client arrives for a scheduled bone marrow aspiration and biopsy. The nurse explains to the client that which position(s) is best for accessing the posterior iliac crest for aspiration? Select all that apply. - Supine - Lateral decubitus - Trendelenburg - Prone - Legs extended chapter 21

- Lateral decubitus - Prone

A client with severe anemia asks, "What happens if my bone marrow cannot produce sufficient numbers of red blood cells due to my bleeding?" Which response(s) by the nurse correctly identifies the organs that can resume hematopoiesis? Select all that apply. - Liver - Spleen - Gall bladder - Pancreas - Kidneys chapter 21

- Liver - Spleen

Blood-borne cancerous cells have recently spread from a woman's primary pancreas tumor to her bones. Which components of the woman's immune system are likely to be directly involved in the attempt to eradicate the potential metastasis? Select all that apply. - Mast cells - Macrophages - T lymphocytes - Natural killer (NK) cells - B lymphocyte cells chapter 6

- Macrophages - T lymphocytes - Natural killer (NK) cells - B lymphocyte cells

The nurse is caring for a client with small cell lung carcinoma who has developed syndrome of inappropriate antidiurectic hormone (SIADH). What should the nurse incorporate into the plan of care? Select all that apply. - Frequent repositioning - Restriction of sodium in the diet - Maintenance of fluid restriction - Daily weights - Administration of hypotonic intravenous solutions chapter 6

- Maintenance of fluid restriction - Frequent repositioning - Daily weights

Which situation would be classified as a complication of a disease or outcome from the treatment regimen. Select all that apply. - Burning, intense incision pain following surgery to remove a portion of colon due to intestinal aganglionosis - Massive pulmonary emboli following diagnosis of new onset atrial fibrillation - Development of pulmonary pulmonary fibrosis following treatment with bleomycin, an antibiotic chemotherapy agent used in treatment of lymphoma - loss of short-term memory in a client diagnosed with alzheimer disease - gradual deterioration in ability to walk unassisted for a client diagnosed with parkinson disease chapter 1

- Massive pulmonary emboli following diagnosis of new onset atrial fibrillation - Development of pulmonary pulmonary fibrosis following treatment with bleomycin, an antibiotic chemotherapy agent used in treatment of lymphoma

The nurse is caring for a client with multiple myeloma. What aspects of care should the nurse prioritize? Select all that apply. - Position in high Fowler's - Monitor urine output - Encourage increased fluid intake - Restrict to a soft diet - Assist for ambulation chapter 24

- Monitor urine output - Encourage increased fluid intake - Assist for ambulation

The ICU nurse is concerned with her client's arterial blood gas (ABG) results-especially the pH 7.30; and PCO2 49 mmHg (6.52 kPa). The nurse interprets these ABG results and assesses her client for which clinical manifestations of respiratory acidosis? Select all that apply. - Hyperactive deep tendon reflexes - Muscle twitching - Numbness in fingers and toes - Complaints of paresthesia sensations around lips/mouth - Headache with complaints of blurred vision chapter 8

- Muscle twitching - Headache with complaints of blurred vision

A client visited his health care provider after finding an enlarged lymph node along the mediastinal border. The health care provider may be alerted to a possible diagnosis of Hodgekin lymphoma based on the client having which other clinical manifestations? Select all that apply. - Joint swelling - Unexplained pruritus - Yeast infection in the mouth - Sore throat with pustules on tonsils - Night sweats chapter 24

- Night sweats - Unexplained pruritus

The nurse is caring for a 45-year-old client undergoing radiotherapy of the mediastinal nodes due to lymphoma. Which intervention should the nurse prioritize. Select all that apply - Performing respiratory assessment - Assessing for fatigue - Avoiding invasive procedures - Performing neurologic assessment - Assessing for urinary retention chapter 3

- Performing respiratory assessment - Assessing for fatigue - Avoiding invasive procedures

A 41-year-old woman diagnosed with multiple sclerosis (MS) is sharing her story with members of an MS support made up of people recently diagnosed. Which aspects of her health problem should the woman warn others to expect at some point in the progression of the disease? Select all that apply. - Progressive loss of visual activity - Debilitating fatigue - Loss of mental acuity - Shuffling gait - Gradual development of resting tumor chapter 15

- Progressive loss of visual activity - Debilitating fatigue - Loss of mental acuity

The nurse suspects a spinal cord injury client is developing autonomic dysreflexia. Which assessment findings would confirm the development of this complication? Select all that apply. - Pulse rate 49 - BP 180/98 - Skin covered with macular rash - Complains of a pounding headache - Cold-cyanotic lower legs

- Pulse rate 49 - BP 180/98 - Complains of a pounding headache

A nurse is caring for a client with post-traumatic stress distress disorder (PTSD). Which behavior would the nurse expect the client to manifest? Select all that apply - Sleep disturbances with vivid nightmares - Disregard for personal safety - Relunctance to participate in group discussions - Difficulty concentrate on tasks - Inability to recall the traumatic events

- Relunctance to participate in group discussions - Difficulty concentrate on tasks - Sleep disturbances with vivid nightmares

A client is brought to the emergency department for an overdose of aspirin, The nurse caring for this client should anticipate which clinical manifestations? Select all that apply. - ABG report: pH 7.50 PCO2 31 mmHg (4.12 kPa), HCO3 level 19 mmol/L - BP 100/72 mmHg - Urine output - 100 mL/hr - Respiratory rate of 40 - Bilateral crackles (fluid) in the lungs chapter 8

- Respiratory rate of 40 - ABG report: pH 7.50 PCO2 31 mmHg (4.12 kPa), HCO3 level 19 mmol/L

The nurse is caring for neonates undergoing treatment for hyperbilirubinemia. Which assessment findings should the nurse prioritize? Select all that apply. - Jaundice - Increase in urine output - Rigidity - Lethargy - Reduction in feeding chapter 23

- Rigidity - Lethargy - Reduction in feeding

The nurse is caring for a client who suffered a spinal injury at T4 several years ago,. The client develops a flushed neck, reports feeling unwell, and has an elevated blood pressure. What are the nurse's priority actions? Select all that apply. - Insert large bore intravenous catheter - Sit client in an upright position - Assess recent urine output - Remove compression stockings - Administer oxygen via face mask chapter 15

- Sit client in an upright position - Assess recent urine output - Remove compression stockings

A client has been diagnosed with multiple sclerosis. What should the nurse include in client teaching to help reduce the frequency of exacerbations? Select all that apply. - Do not take over-the counter medications - Stay up-to-date with your vaccines - Avoid extremes of environmental temperature - Stay out of direct sunlight - Avoid stressful situations chapter 15

- Stay up-to-date with your vaccines - Avoid extremes of environmental temperature - Avoid stressful situations

While taking a prenatal history, the nurse should be most concerned about severe teratogenic effects on the fetus if the mother admits to taking which medication prior to finding out she was pregnant? Select all that apply - Tetracycline for acne - Ethyl alcohol ingestion regularly several times per week - Isotretinoin for acne - Over-the-counter cetirizine for seasonal allergies - Warfarin for chronic atrial fibrillation chapter 5

- Tetracycline for acne - Ethyl alcohol ingestion regularly several times per week - Isotretinoin for acne - Warfarin for chronic atrial fibrillation

A clinical nurse specialist is interested in developing a research study focused on clients living with the sequelae of ischemic stroke. Which clients should the nurse include? Select all that apply. The nurse should include clients with: - vision changes - aspiration pneumonia - falls - unilateral weakness - dysphagia (difficulty swallowing) chapter 1

- vision changes - dysphagia (difficulty swallowing) - unilateral weakness

A nurse is investigating the epidemiologic factors influencing breast cancer for women in a population. What information should the nurse include? Select all that apply. - The geographic location of women diagnosed with breast cancer - The currently known incident and prevalence for this population - How cancer cells divide and multiple in breast tissue - The number of women whose diagnosis was assisted by the use of mammography - The age of women at the time of diagnosis chapter 1

- The age of women at the time of diagnosis - the geographic location of women diagnosed with breast cancer

A community health nurse is teaching a group of recent graduates about the large variety of factors that influence an individual's health or lack thereof. The nurse is referring to the Healthy People 2020 report from the U.S. Department of Health and Human services as a teaching example. Of the following discussed, which would be considered a determinant of health that is outside the focus of this report? - The client has a family history of cardiovascular disease related to hypercholesterolemia and remains noncompliant with the treatment regime - The client has a good career with exceptional preventative health care benefits - The client has a diverse background by being of Asian and native American/first nation descent and practices various alternative therapies to minimize effects of stress - the client lives in an affluent, clean, suburban community with access to many health care facilities - Chapter 1

- The client has a family history of cardiovascular disease related to hypercholesterolemia and remains noncompliant with the treatment regime

A 77-year-old man is a hospital inpatient admitted for exacerbation of his chronic obstructive pulmonary disease (COPD), and a respiratory therapist (RT) is assessing the client for the first time. Which aspect of the client's current state of health would be best characterized as a symptom rather than a sign? - The RT hears diminished breath sounds to the client's lower lung fields bilaterally - The client notes that he has increased work of breathing when lying supine - The client's respiratory rate is 31 breaths per minute - The client's oxygen saturation is 83% by pulse oximetry chapter 1

- The client notes that he has increased work of breathing when lying supine

A 77-year-old male client with a diagnosis of stomach cancer has been found to have metastases in his liver. The client and his family are surprised at this turn of events, stating that they don't see how he could have developed cancer in his liver. Which fact would underlie the reply that the care team provides? - The proximity of the liver to the stomach allows for direct spread of cancerous cells due to a lack of contact inhibition - Hepatic stromal tissue shares characteristics with cancerous cells, including lack of anchorage dependence - The parenchymal tissue of the liver is particularly susceptible to secondary malignancies - The portal circulatory system brings venous blood from the GI tract into the liver chapter 6

- The portal circulatory system brings venous blood from the GI tract into the liver

The health care provider is ordering a complete blood count (CBC). What information should the nurse expect to be able to gather from the results? Select all that apply - Presence of thrombocytopenia - If the client has anemia - If the client is deficient in clotting factors - The presence of leukocytosis - If there are common vitamin deficiencies chapter 21

- The presence of leukocytosis - Presence of thrombocytopenia - If the client has anemia

Following a biopsy, a 54 year-old man has been diagnosed as having a benign neoplastic tumor. Which characteristics most likely applies to his tumor? - It has a rapid rate of growth and can induce ischemia - The tumor may secrete hormones, cytokines - The tumor is poorly approximated and has the potential to break loose - The well-differentiated, neoplastic cells are clustered together in a single mass chapter 6

- The well-differentiated, neoplastic cells are clustered together in a single mass

A client with leukemia begins hemorrhaging from every orifice. The physician is concern that the client has developed disseminated intravascular coagulation (DIC). The nurse should anticipate which order to be prescribed for this client? Select all that apply. - Administer IV ketorolac STAT - Give aspirin twice/day - Place in reverse isolation - Transfuses fresh frozen plasma - Transfuse 2 units of platelets chapter 22

- Transfuses fresh frozen plasma - Transfuse 2 units of platelets

A newly diagnosed HIV-positive adolescent has blood work drawn that includes a CD8+ T-cell count. The nurse knows that which functions of CD8+ T cells listed below will assist the adolescent's immune system in fighting off the viral attack? Select all that apply - Cause allergens to surround the virus - Release of destructive enzymes - Booster antigen-antibody response - Trigger intracellular programmed death - Removal of foreign material from lymph before it enters the blood chapter 11

- Trigger intracellular programmed death - Release of destructive enzymes

The nurse is caring for a client with sepsis who has developed disseminated intravascular coagulation (DIC). Which assessments should the nurse prioritize? Select all that apply. - Level of consciousness - Oxygen saturation - Platelet count - Urine output - Weight chapter 22

- Urine output - Level of consciousness - Oxygen saturation

The nurse is performing a passive range-of-motion exercises with a client who suffered an ischemic stroke 2 weeks ago and has hemiparesis. The client says, "What is the point of doing these exercises if I will never be able to use that arm again?" What should the nurse include when responding to the client. Select all that apply - We want to ensure your shoulder joint remains flexible while you recover - You may still regain use of this arm, and these exercises may help with this recovery - Doing these exercises helps to prevent blood clots from forming in the arteries of your arm - These exercises help to prevent painful shoulder complications after a stroke - It is important for you to keep a positive outlook as it will help with your recovery chapter 16

- We want to ensure your shoulder joint remains flexible while you recover - You may still regain use of this arm, and these exercises may help with this recovery - These exercises help to prevent painful shoulder complications after a stroke

Following head injury from a fall, the child's CT scan reveals a moderate brain injury contusion. Which manifestations will the nurse more than likely assess on this child that supports this diagnosis? Select all that apply. - Weakness or slight paralysis affecting on side of the body - Coma with total paralysis - Nuchal rigidity - Periods of unconsciousness - Aphasia at times chapter 16

- Weakness or slight paralysis affecting on side of the body - Periods of unconsciousness - Aphasia at times

While volunteering in an HIV clinic in a big city, the nurse notices a new mom and her 6-month old child in the waiting room. Upon assessing the infant for possible HIV infection, the nurse will be assessing for which clinical manifestations of HIV infection? Select all that apply - Weighing him to determine if he is gaining 1.5 to 2 pounds/month - History of repeated episodes of bacterial pneumonia and ear infections - Lack the coordination to play with toys/stuffed animals - Listlessness and poor eye contact - Observing to see if he can roll over from back to stomach chapter 12

- Weighing him to determine if he is gaining 1.5 to 2 pounds/month - History of repeated episodes of bacterial pneumonia and ear infections - Lack the coordination to play with toys/stuffed animals - Listlessness and poor eye contact

A female client arrives at the clinic reporting fatigue that is exhausting, bruising on the skin, and bleeding from the gums. Which diagnostic result(s) or history finding(s) correlates with a diagnosis of plastic anemia? - White blood cell count - Completed chemotherapy for leukemia in the past year - Platelet count - Hemoglobin level of 12.5 g/dL - Recent bout with mononucleosis viral infection

- White blood cell count - Completed chemotherapy for leukemia in the past year - Platelet count - Recent bout with mononucleosis viral infection

The nurse is caring for a client who is suspected to have multiple myeloma. What findings will the nurse use as evidence the client has multiple myeloma? Select all that apply. - X-rays demonstrate bone loss - 20% plasma cells in bone marrow biopsy - Bence Jones proteins in urine - Hyperkalemia - Erythrocytosis chapter 24

- X-rays demonstrate bone loss - 20% plasma cells in bone marrow biopsy - Bence Jones proteins in urine

A member of the health care team is researching the etiology and pathogenesis of a number of clients who are under his care in a hospital context. Which aspect of client's situations best characterizes pathogenesis rather than etiology? - A client who was admitted with the effects of methyl alcohol poisoning - a client with multiple skeletal injuries secondary to motor vehicle accident - a client who is has increasing serum ammonia levels due to liver cirrhosis - a client who has exposed to the mycobacterium tuberculosis bacterium chapter 1

- a client who is has increasing serum ammonia levels due to liver cirrhosis

The nurse is caring for a client with arterial insufficiency of the left leg with gangrenous wounds on the second and third toes. What characteristics of the wounds should the nurse evaluate as expected? Select all that apply. - Edema to the left lower leg - diminished pulse strength - moist wound with evidence of liquefaction - darkened appearance of affected tissue - a clear demarcation between the healthy and affected tissue - atrophy to the affected toes chapter 3

- diminished pulse strength - darkened appearance of affected tissue - a clear demarcation between the healthy and affected tissue - atrophy to the affected toes

A community health nurse is discussing preventative vaccines with a group of primigravda women. Which cancer-causing virus(es) should the nurse discuss that have a vaccine to prevent developing the disease? Select all that apply. - human papillomavirus (HPV) - meningitis - hepatitis B - chlamydia - zika virus chapter 6

- hepatitis B - human papillomavirus (HPV)

Which clients are at increased risk for developing secondary hypogammaglobulinemia, a secondary humoral immunodeficiency? Select all that apply. - child allergic to furs, grass, and trees - malnourished child who drinks cow's milk - client with a history of seizures controlled by antiepileptic - client with burns covering more than 45% of the body - client with a history with anxiety and depression chapter 12

- malnourished child who drinks cow's milk - client with a history of seizures controlled by antiepileptic - client with burns covering more than 45% of the body

The nurse is caring for a client with traumatic brain injury and increased intracranial pressure. What findings should the nurse report due to the risk of increasing intracranial pressure? Select all that apply - pH of 7.25 - PaCO2 of 28 mm Hg (3.72 kPa) - potassium of 3.3 mEq/L (3.3 mmol/L) - PaO2 of 70 mmHg (9.31 kPa) - PaCO2 of 50 mm Hg (6.65 kPa) chapter 16

- pH of 7.25 - PaO2 of 70 mmHg (9.31 kPa) - PaCO2 of 50 mm Hg (6.65 kPa)

When educating a client who has tested positive for human T-cell leukemia virus-1 (HTLV-1), what mode(s) of transmission should the nurse discuss to prevent the spread? Select all that apply. - sexual intercourse - kissing a person on the forehead - blood by sharing needles - sneezing (releasing droplets) - infants through breast milk chapter 6

- sexual intercourse - infants through breast milk - blood by sharing needles

The nurse working in a pediatric office is scheduled to assess a female adolescent diagnosed with neurofibromatosis (NF) Type 1. During this assessment, the nurse should be assessing the teenager for which clinical manifestations of NF-1? Select all that apply - irregular menstrual periods - hearing loss - speech impediments - reports of having a hard time concentrating in school - severe scoliosis chapter 5

- speech impediments - reports of having a hard time concentrating in school - severe scoliosis

A male child was born with Klinefelter syndrome. As an adolescent, the nurse will assess which clinical manifestations of Klinefelter syndrome? Select all that apply. - tall stature out of proportions - enlarged breast tissue - sparse facial and pubic hair - higher than average linguistic skills - severe intellectual activity chapter 5

- tall stature out of proportions - enlarged breast tissue - sparse facial and pubic hair

As of Nov. 1, 2012, there were a total of 10 confirmed cases of hantavirus infection in people who were recent visitors (mid-June to end of August, 2012) to Yosemite National Park. Three visitors with confirmed cases died. Health officials believe that 9 out of the 10 cases of hantavirus were exposed while staying in Curry Village in the Signature Tent Cabins. This is an example of: - the incidence of people who are at risk for developing hantavirus while staying in Yosemite National Park what the anticipated mortality rate would be if a family of five were planning to vacation in Yosemite National Park - the low rate of morbidity one can expect while traveling to Yosemite National Park - the prevalence of hantavirus one can anticipate if he or she is going to vacation in Yosemite National Park chapter 1

- the incidence of people who are at risk for developing hantavirus while staying in Yosemite National Park

A young client has just been diagnosed with xeroderma pigmentosum, when teaching this family about this disease. the nurse should emphasize which teaching points? Select all that apply. - the best time for this family to go to the beach is in the fall/winter months - wash hands thoroughly when working in the garden to prevent infection - apply antibacterial ointment to any break in the skin and cover wounds with bandages - wear long sleeves, long pants, gloves, a hat, sunglasses with side shields, and sunscreen while outdoors - the best time to allow the child to play outside is in the evening hours after the sun goes down chapter 3

- wear long sleeves, long pants, gloves, a hat, sunglasses with side shields, and sunscreen while outdoors - the best time to allow the child to play outside is in the evening hours after the sun goes down

The neuroscience nursing unit has developed a set of step-by-step directions of what should occur if a nursing assessment reveals the client may be exhibiting clinical manifestations of a cerebrovascular accident (CVA). Which statement about clinical practice guidelines are accurate? Select all that apply. - Once developed, practice guidelines only need to be reviewed if a national committee sends out an update on new research - step-by-step guidelines are usually developed and based primarily on "how it has always been done before" - when developing a CVA set of step-by-step directions, the nursing unit should ask for assistance from experts in the neuroscience field. The potential users of the guidelines should pilot test it for further feedback - A meta-analysis could be utilized to combine evidence from different studies to produce a more accurate diagnostic method - the development of evidence-based guidelines require a research review from different studies to develop the most accurate diagnostic method to implement chapter 1

- when developing a CVA set of step-by-step directions, the nursing unit should ask for assistance from experts in the neuroscience field. The potential users of the guidelines should pilot test it for further feedback - A meta-analysis could be utilized to combine evidence from different studies to produce a more accurate diagnostic method - the development of evidence-based guidelines require a research review from different studies to develop the most accurate diagnostic method to implement

The nurse is counseling a client who is going for major surgery and must stop taking aspirin to reduce the risk for bleeding. The nurse knows aspirin decreases platelet activity for the duration of the platelet's lifespan. How many days prior to the surgery should the nurse tell the client to stop taking aspirin? - 10 - 2 - 14 - 5 chapter 21

10

During a period of extreme excess fluid volume, a renal dialysis client may be administered which type of IV solution to shrink the swollen cells by pulling water out of the cell? - 0.9% sodium chloride - 3% sodium chloride - 5% dextrose and water - Lactated Ringer's solution chapter 8

3% sodium chloride

Which client would be considered to be in the latent period of HIV infection? - 33-year-old heroin drug abuser who has numerous enlarged lymph nodes in his axilla and cervical region for the past 4 months - 16-year-old prostitute who has open sores on her labia that drain purulent secretions - 45-year-old alcohol abuser who is reporter excessive vomiting of blood that started 2 weeks ago - 24-year-old college student who has developed a chronic cough that will not go away, even after taking two course of antibiotics chapter 12

33-year-old heroin drug abuser who has numerous enlarged lymph nodes in his axilla and cervical region for the past 4 months

Which client has an absolute neutrophil count (ANC) that is critically low and the standard of care would recommend placement on neutropenic precautions? - 37-year-old client with leukemia being treated with chemotherapy with ANC of 400 - 75-year-old client with renal failure receiving epogen for anemia with hemoglobin level of 9.7 g/dL - 65-year-old client with prostate cancer receiving radiation therapy with neutrophil count of 2,000 - Client on long-term steroids for rheumatoid arthritis with WBC of 7000 chapter 24

37-year-old client with leukemia being treated with chemotherapy with ANC of 400

When talking about the lifespan of various blood components, the student should know that once a neutrophil moves into tissue, it lives approximately how long? - 12 hours - 24 hours - 2 days - 4 days chapter 21

4 days

Of the following situations, which one would be an example of a maladaptive cellular change? - 31-year-old marathon runner who has developed hypertrophied myocardial cells - 54-year-old female who has developed ovarian atrophy following loss of estrogen stimulation during menopause - 18-year-old body builder who has developed extremely large pectoral muscles following years of weight lifting - 44-year-old male with a 60 pack/year smoking history who ws diagnosed with a histological grade-3 lung cancer chapter 3

44-year-old male with a 60 pack/year smoking history who ws diagnosed with a histological grade-3 lung cancer

Which client is at high risk for developing dilated cardiomyopathy? - 4-year-old child born with cerebral palsy and confined to a wheelchair - 17-year-old with a diving injury resulting in paraplegia - 78-year-old Alzheimer client who received a third-degree burn following an oven fire - 44-year-old noncompliant female who forgets to take her hypertensive medication chapter 3

44-year-old noncompliant female who forgets to take her hypertensive medication

A fifth-grade elementary student asks the school nurse how much blood is in an entire body. The nurse should respond that the average grown-up adult has: - 3 to 4 quarts of blood in the body - 2 to 4 cups of blood in the body - 3 pints of blood total - 5 to 6 liters of blood throughout the body chapter 21

5 to 6 liters of blood throughout the body

Which client would have a very poor response related to tissue regeneration of his or her injured area? - 33-year-old athlete undergoing surgery to repair a torn MCL in the right knee - 21-day- old infant undergoing a diaphragmatic hernia repair - 73-year-old female who is undergoing lithotripsy for kidney stones - 54-year-old male who had a massive MI 4 days ago and came to the ED today for treatment chapter 9

54-year-old male who had a massive MI 4 days ago and came to the ED today for treatment

A medical student is familiarizing herself with recent overnight admissions to an acute medical unit of a university hospital. Which client would the student recognize as least likely to have a diagnosis of antiphospholipid syndrome in his or her medical history? A 90-year-old female resident of a long-term care facility who has been experiencing transient ischemic attacks - A 21-year-old male with a diagnosis of cellulitis and suspected endocardititis secondary to intravenous drug use - A 30-year-old female with a diagnosis of left leg deep vein thrombosis and pulmonary embolism - A 66-year-old obese male with left-sided hemiplegia secondary to a cerebrovascular accident chapter 22

A 21-year-old male with a diagnosis of cellulitis and suspected endocardititis secondary

In which client would diagnostic investigations be least likely to reveal increased thrombopoietin production? - A 66-year old woman with a diagnosis of lung cancer with bone metastasis - A 21-year-old woman awaiting bone marrow transplant for myelogenous leukemia - An 81-year-old woman with a diagnoses of rheumatoid arthritis and failure to thrive - A 55-year-old man with dehydration secondary to Crohn disease chapter 22

A 21-year-old woman awaiting bone marrow transplant for myelogenous leukemia

Which client of a primary care physician would not require extra screening for cancer? - A 48-year-old man who takes immunodepressant drugs following a kidney transplant - A 50-year-old who is obese and has a low-fiber, high-fat diet - A 51-year-old woman whose grandmother died of breast cancer - A 38-year-old female with Down syndrome and congenital scoliosis chapter 6

A 38-year-old female with Down syndrome and congenital scoliosis

In which individuals would a clinician most suspect multiple myeloma as a diagnosis? - A 68-year-old former coal miner who has white blood cell levels exponentially higher than normal ranges - An 81-year-old male resident of a long-term care home who has an uncommon bacterial pneumonia and who is unable to produce a fever A 70-year-old woman whose blood work reveals large numbers of immature granulocytes - A 40-year-old man who has had 3 broken bones over the past 6 months and whose serum calcium and creatinine levels are elevated chapter 24

A 40-year-old man who has had 3 broken bones over the past 6 months and whose serum calcium and creatinine levels are elevated

The nurse is caring for a group of postoperative clients. Who should the nurse prioritize as having the greatest risk for infection at the surgical site? - A 22-year-old 2 days post-appendectomy - A 15-year-old 1 day post-tonsillectomy - A 45-year-old with external fixation of fractured tibia - A 65-year-old awaiting pacemaker insertion chapter 9

A 45-year-old with external fixation of fractured tibia

Which situation can be best characterized as an example of passive immunity? - A 9 year old boy is immune to chickenpox after enduring the infection 1 year previous - A 6 week old infant receives antibodies from his mother's breast milk - An 8 year old girl recovers from a respiratory infection after intravenous antibiotic treatment - A 6 month old infant receives his scheduled immunization against measles, mumps, and rubella chapter 11

A 6 week old infant receives antibodies from his mother's breast milk

Which client is most likely to be a candidate for a thymus transplant as the treatment of choice to reconstitute T-cell immunity? - A 6-year-old boy whose pre-B cells are incapable of transplantation to normal B cells - A 9-year-old girl who has developed secondary cell-mediated immunodeficiency following a viral infection - A 7-year-old boy with diagnosis of thymic aplasia whose blood work indicates absence of T cells - A 12-year old girl with a history of epilepsy and low IgG levels secondary to phenytoin use chapter 12

A 7-year-old boy with diagnosis of thymic aplasia whose blood work indicates absence of T cells

Which client would be most likely to be experiencing an increase in renal erythropoietin production? - A 70-year-old woman admitted with dehydration secondary to an overdose of her potassium-wasting diuretic - A 71-year-old smoker admitted to hospital with exacerbation of his chronic obstructive pulmonary disease (COPD) - A 21-year-old man with acute blood loss secondary to a motor vehicle accident 3 hours prior - A 68-year-old man with a long-standing diagnosis of polycythemia vera chapter23

A 71-year-old smoker admitted to hospital with exacerbation of his chronic obstructive pulmonary disease (COPD)

Which individuals would most likely possess normal plasma synthesis and fully differentiated myeloid and lymphoid cells? - A 58-year-old female with HIV and multiple myeloma - A 78-year-old make who has been diagnosed with chronic lymphocytic leukemia (CLL) - A 7-year-old boy with a diagnosis of acute lymphocytic leukemia (ALL) - A 70-year-old male who has acute myelogenous leukemia (AML) chapter 24

A 78-year-old make who has been diagnosed with chronic lymphocytic leukemia (CLL)

Which individual situation listed below best exemplifies the process of innate immunity? - A client who has experienced rejection of a donor live after transplantation - A child who has experienced heat and swelling of his skinned knuckles - A client whose blood work indicates increased antibody titers during an acute illness - An adult who complains of itching and is sneezing because he is allergic to pollen chapter 11

A child who has experienced heat and swelling of his skinned knuckles

Which scenario best describes an example of infection originating with a fomite? - A nurse with positive tuberculin screening test (PPD) after admitting a client diagnosed with tuberculosis (TB) - A woman who contracted Lyme disease after a tick bite - A man who has been diagnosed with trichinosis after eating undercooked pork - A client who contracted hepatitis C through sharing contaminated syringe with an infected person chapter 10

A client who contracted hepatitis C through sharing contaminated syringe with an infected person

Two health care workers are comparing the etiology and incidence of multifactorial inheritance disorders and single-gene disorders. Which statement best captures the relationship between the two types of genetic disorders? - A couple with a child with a multifactorial disorder have a higher risk of having another with the same disorder - Multifactorial disorders and a single-gene disorders can both be predicted quite accurately - Multifactorial disorders are more likely to involve multiple organs - Multifactorial disorders manifest themselves at birth chapter 5

A couple with a child with a multifactorial disorder have a higher risk of having another with the same disorder

A 22-year-old female college student is shocked to receive a diagnosis of myasthenia gravis. What are the etiology and most likely treatment for her health problem? - Autoimmune destruction of skeletal muscle cells; treatment with intensive physical therapy and anabolic steroids - Cerebellar lesions; surgical and immunosuppresive treatment - Excess acetycholinesterase production; treatment with thymectomy - A decline in functioning acetycholine receptors; treatment with corticosteroids and intravenous immunoglobulins chapter 15

A decline in functioning acetycholine receptors; treatment with corticosteroids and intravenous immunoglobulins

A client with a diagnosis of atrophic gastritis and consequent pernicious anemia is receiving high oral doses of vitamin B12. Following completion of his treatment, which lab finding demonstrates improvement in this condition? - Decreased free heme levels - Increased serum bilirubin - Increased folic acid levels - A decrease in mean corpuscular volume chapter 23

A decrease in mean corpuscular volume

Which individual would be most likely to experience global ischemia to his or her brain? - A man who has entered cardiogenic shock following a severe myocardial infarction - A woman who has been admitted to the emergency department with a suspected intracranial bleed - A woman who is being brought to the hospital by ambulance following suspected carbon monoxide poisoning related to a faulty portable heater - A male client who has just had an ischemic stroke confirmed by CT on his head chapter 16

A man who has entered cardiogenic shock following a severe myocardial infarction

The nurse knows which statement listed below is accurate regarding the functions and nature of cytokines relative to variety of pathologies? - Cytokine production is constant over time, but effects are noted when serum levels cross a particular threshold - Cytokine actions are self-limiting, in that activation of one precludes activation of other cytokines with similar actions - Most cytokines are produced by granular leukocytes, and different cells are capable of producing the same cytokine - A particular cytokine can have varied effects on different systems, a fact which limits their therapeutic use - chapter 11

A particular cytokine can have varied effects on different systems, a fact which limits their therapeutic use

The nurse evaluates the ECG of a client who has developed hypokalemia seconary to diuretic use. Which manifestation of hypokalemia does the nurse anticipate on the ECG? - A prominent U wave and flattened T wave - Irregular heart rate and a peaked T wave - A low T wave and an absent P wave - A narrow QRS complex and an absent U wave chapter 8

A prominent U wave and flattened T wave

A client is experiencing muscle atrophy following 2 weeks in traction after a motor vehicle accident. Which factor has most likely contributed to the atrophy of the client's muscle cells? - Denervation of the affected muscles during the time of traction - Reduced oxygen consumption and cellular function that ensures muscle cell survival - a reduction of skeletal muscle use secondary to the traction treatment - high levels of insulin and IGF-1 in the client's blood during immobilization chapter 3

A reduction of skeletal muscle use secondary to the traction treatment

When counseling a male client with suspected HIV, the nurse informs him that if the enzyme-linked immunosorbent assay (ELISA) comes back positive, then: - If the second test, the Western blot, returns negative, he has not developed a case of full-blown AIDS - No further testing is required since this confirms HIV infection - A second test known as the Western blot assay will be ordered to confirm positive HIV status - He will be sent to an infectious disease physician for a tissue biopsy to confirm infection chapter 12

A second test known as the Western blot assay will be ordered to confirm positive HIV status

A middle school student is scheduled to receive booster immunizations and the father asks the nurse why the booster is necessary. What characteristic of the adaptive immune system listed below would provide the rationale for the nurse's response? - Some antibodies require a repeat of the primary immune response - Antigen receptors on CD4+ cells require multiple exposures separated by time - A secondary response cause a sharp rise in antibody levels - Some antibodies have a duration measured in months rather than years chapter 11

A secondary response cause a sharp rise in antibody levels

A 24-year-old woman presents with fever and painful, swollen cervical lymph nodes. Her blood work indicates neutrophilia with a shift to the left. She most likely has: - A severe bacterial infection - A severe fungal infection - A mild viral infection - A mild parasitic infection chapter 9

A severe bacterial infection

A hospital client has a large, superficial wound on her elbow that was the result of shearing action when she was moved up in her bed. The client's husband mentions that the wound looks infected and irritated since it is completely red. Which response by the nurse would be inappropriate? - Even though it is red, it doesn't mean that the wound is infected - The red areas show that there is enough circulation to facilitate healing - Those are fresh blood vessels that are a sign of healthy healing - A thin sheet of blood clotting is actually desirable and not a sign that your wife's wound is infected chapter 9

A thin sheet of blood clotting is actually desirable and not a sign that your wife's wound is infected

A 26 year-old female is resting after 1-minute episode during which she lost consciousness while her muscles contracted and extremities extended. This was followed by her rhythmic contraction and relaxation of her extremities. On regaining consciousness, she found herself to have incontinent of urine. What has the woman most likely experienced? - A myoclonic seizure - A complex partial seizure - An absence seizure - A tonic-clonic seizure chapter 16

A tonic-clonic seizure

A 41-year-old female with a family history has had a baseline mammogram, She states that she performs monthly self breast exams but has really has a hard time evaluating her lumps since she has numerous cysts. At her annual mammogram, the technician views a suspicious area and refers her to the radiologist. She asks the nurse in the office, "How can a lump appear so quickly?" The nurse's response is based on which principle? - Many tumor cells never leave the M phase of the cell cycle - If the breast has a lot of cysts, then the fluid within those sacs makes it hard to feel the hard lumps of a cancer - Cancer cells are undifferentiated and come in various shapes and sizes - A tumor is undetectable until it has doubled 30 times and contains at least 1 billion cells chapter 6

A tumor is undetectable until it has doubled 30 times and contains at least 1 billion cells

During a flu shot clinic, one of the questions the student nurse asks relates to whether the patient has had Guillain-Barre? syndrome in their medical history. The patient asks, "What is that?" How should the nursing student reply? - Influenza-like illness where you had fever and chills for 2 to 3 days after your last flu shot - A degenerative disease where you have trouble walking without the help of a cane or walker - A type of paralysis that affects movement on both sides of the body, it may even involve the respiratory muscles - Swelling of your arm where you got your flu shot and maybe your eyes and lips had some swelling as well chapter 15

A type of paralysis that affects movement on both sides of the body, it may even involve the respiratory muscles

Which pregnant women has most likely encountered the greatest increase in the risk that her child will have a fetal anomaly? - A woman with diagnoses of syphilis and cirrhosis of the liver. - A woman with chronic obstructive pulmonary disease (COPD) and tuberculosis - A woman with diagnoses of insulin-dependent diabetes mellitus and peripheral neuropathy - A woman who has herpes simplex and who has recently recovered from endocarditis chapter 5

A woman who has herpes simplex and who has recently recovered from endocarditis

A client with a gastrointestinal bleed secondary to alcohol abuse and a hemoglobin level of 5.8 g/dL (58(g/L) has been prescribed a transfusion of packed red blood cells. The client possesses type B antibodies but lacks type D antigens on his red cells, Transfusions of which blood type would be least likely to produce a transfusion reaction? - A+ - B- - B- - A- chapter 23

A-

A 32-year-old woman present at her health clinic complaining of weakness, feeling of abdominal fullness, 6-month history of fatigue and night sweats. She added a multivitamin with iron and some extra meat and leafy greens to her diet but has not experienced an increase in energy. Upon assessment, her spleen was noted to be enlarged. Which diagnosis is most likely associated with her manifestations? - Infectious mononucleosis - Stage A Hodgkin disease - Chronic lymphocytic leukemia - Accelerated chronic myelogenous leukemia chapter 24

Accelerated chronic myelogenous leukemia

Which phenomenon would be least likely to result in activation of the complement system? - Recognition of an antibody bound to the surface of a microbe - Increase tissue blood flow and capillary permeability to fluids/proteins can leak into the area - Activation of toll-like receptors (TLRs) on complement proteins - Direct recognition of microbial proteins chapter 11

Activation of toll-like receptors (TLRs) on complement proteins

A student makes the statement to a colleague, "Blood plasma is essentially just a carrier for the formed cells like red blood cells and white blood cells." What would be the most accurate response to this statement? - Actually, plasma plays a significant role in nutrient and waste transport - Not really/ Plasma also contributes to the processes of protein synthesis and hematopoiesis - That's not really true. Plasma is crucial in the immune and inflammatory responses - Actually, plasma is integral to the proper function of the liver and maintenance of acid-base balance chapter 21

Actually, plasma plays a significant role in nutrient and waste transport

A client is suspected of having bacterial meningitis. Which action should the nurse complete first? - Complete neurological assessment - Administer antipyretics - Administer antibiotics - Prepare for lumbar puncture chapter 16

Administer antibiotics

The nurse is caring for a client who meets the physical assessment criteria for sepsis, but the specimen cultures are negative for a causative organism. What action should the nurse take next? - Continue to monitor signs and symptoms - Contact the health care provider - Administer the prescribed antibiotics - Recollect specimen cultures chapter 10

Administer the prescribed antibiotics

A 72-year old client being treated for rheumatoid arthritis is admitted with suspected ehrlichiosis. What should the nurse implement as a treatment priority? - Isolation of the client using contact precautions - Collection of the appropriate blood and fluid cultures - Administration of empirically chosen antibiotics - Isotonic intravenous fluids at a minimum rate of 200 mL/hr chapter 10

Administration of empirically chosen antibiotics

A 45-year-old woman with a diagnosis of shingles is experiencing an acute onset of severe neuropathic pain. Which stage of Selye's characterization of stress response is the woman most likely experiencing at the moment? - Exhaustion - Resistance - Alarm - General Adaptation Syndrome (GAS) chapter 7

Alarm

As part of a screening program for prostate cancer, men at a senior citizens' center are having their blood levels of prostate-specific antigen (PSA) measured. Which of the following statements would best characterize a high positive predictive value but a low negative predictive value for this screening test? A. All of the men who had high PSA levels developed prostate cancer; several men who had low PSA levels also developed prostate cancer. B. All of the men who had low PSA levels were cancer-free; several men who had high levels also remained free of prostate cancer. C. Men who had low PSA levels also displayed false-positive results for prostate cancer; men with high levels were often falsely diagnosed with prostate cancer. D. The test displayed low sensitivity but high specificity. chapter 1

All of the men who had high PSA levels developed prostate cancer; several men who had low PSA levels also developed prostate cancer.

The nurse is preparing for a bone marrow transplant. The client asks, "How long will it take for the new marrow to make the blood cells?" What is the nurse's best response? - The stem cells that are injected will take the normal amount of time to create the cells as if it were your own marrow - Although it can vary, we will administer medication that will stimulate the bone marrow to create the cells - Because we place the donated marrow directly into your iliac crest, the marrow will start making cells right away - The new marrow will not be creating blood cells until it has homed to the appropriate areas and matured chapter 11

Although it can vary, we will administer medication that will stimulate the bone marrow to create the cells

Which type of pneumonia is best characterized by an infective agent that produces sputum samples with a peptidoglycan cell wall, expresses endotoxins, replicates readily in broth and on agar, grows in clusters, has pili, and does not stain when exposed to crystal violet? - Mycoplasmal - Bacterial - Viral - Chlamydial chapter 10

Bacterial

Staphylococcus aureus commonly found in the skin, nares, and other body sites of clients without any signs and symptoms of infections is known as which condition? - A parasitic infection - Bacterial colonization - An opportunistic infection - A saprophytic infection chapter 10

Bacterial colonization

When talking about the various types of granulocytes, which granule contains heparin, an anticoagulant? - Neutrophils - Basophils - Eosinophils - Lymphocytes chapter 21

Basophils

A client is admitted for cardioembolic stroke. Which therapy to best prevent recurrence of embolic stroke should the nurse monitor for effectiveness? - Correction of dyslipidemia - Blood pressure management - Anticoagulation therapy - Antirrhythmic therapy chapter 16

Anticoagulation therapy

An 81-year-old female client in a subacute medical unit of a hospital has developed an oral Candida albicans infection. Which phenomena would the client's nurse suspect as a key contributing factor to her infection? - The airborne communicability of yeast and molds and subsequent inhalation - The moist and temperature-suited oral environment of the client's mouth - Antibiotic therapy that eliminated normal bacterial flora - The ability of fungi to remain latent until the host reaches an immunocompromised state chapter 10

Antibiotic therapy that eliminated normal bacterial flora

A client is having an edrophonium test. The client suddenly goes into respiratory arrest. Which priority medication should the nurse be prepared to administer? - Mixed alpha-beta adrenergic agonist - Acetycholinesterase inhibitor - Inhaled beta-2 agonist - Anticholinergic chapter 15

Anticholinergic

Following a spider bite she received while camping, a 20-year-old female presented to the emergency department with rash, edema, and fever and was subsequently diagnosed with serum sickness. Which statement best conveys the physiologic rationale for the broad systemic effects of this event? - Antigen-antibody complexes have been deposited in a variety of locations throughout the body - The woman is experiencing diffuse tissue necrosis as a consequence of an Arthrus reaction - Antibody binding to specific target cell receptors is bringing about a change in cell function - Deposited antibodies are activating her complement system chapter 12

Antigen-antibody complexes have been deposited in a variety of locations throughout the body

A client with a surgical wound has developed excessive granulation tissue extending above the wound edges. Which action should the nurse take? - Contact the health care provider to order wet-to-dry dressings to promote debridement of the tissue - Ask the health care provider about surgical options to remove the excess tissue and promote wound healing - Apply an occlusive dressing to increase moisture in the wound bed as excessive granulation is due to dryness - Take no action; this overgrowth of tissue is referred to as "keloids" and is not amenable to treatment chapter 9

Ask the health care provider about surgical options to remove the excess tissue and promote wound healing

The nurse is caring for a client hospitalized yesterday with acute coronary syndrome. The client also has difficulty dealing with life stressors. On what should the nurse focus as the priority of care? - Teaching the client stress management techniques such as guided imagery and journaling - Encouraging physical activity along with a healthy diet to slow the progression of heart disease - Assessing heart rate and blood pressure and interviewing the client about chest pain - Counseling the client on how to reduce life stressors to decrease the impact on overall health chapter 7

Assessing heart rate and blood pressure and interviewing the client about chest pain

A 47-year-old woman was diagnosed with amyotrophic lateral sclerosis 3 years ago and has experienced a progressive onset and severity of complications. She has been admitted to a palliative care unit due to her poor prognosis. What assessments and interventions should the nursing staff of the unit prioritize in their care? - Assessment of swallowing ability and respiratory status - Regular pain assessment and administration of opioid analgesics as needed - Assessment and documentation of cognitive changes, including confusion and restlessness - Cardiac monitoring and administration of inotropic medications chapter 15

Assessment of swallowing ability and respiratory status

Which statement most accurately conveys an aspect of lymphatic system activity? - B cells and macrophages are released from the bone marrow in their completed state - Leukocytes bypass vascular circulation and are distributed instead by the lymphatic system - Stem cells in the lymph nodes initiate and regulate the process of white cell synthesis - B and T lymphocyte development begins in the bone marrow and ends in the peripheral lymphoid structures chapter 24

B and T lymphocyte development begins in the bone marrow and ends in the peripheral lymphoid structures

Following a bone marrow biopsy, which of the following assessments would indicate the patient is experiencing a complication as a result of this diagnostic procedure? - BP 90/60, heart rate 132, excess bleeding, and hematoma noted at insertion site - Respiratory rate 24, complaining of pain at insertion site - Heart rate regular 64 bpm, temperature 99.6 F, (37.5 C) orally - BP 130/80, oxygen saturation 95%, crackles heard on inspiration chapter 21

BP 90/60, heart rate 132, excess bleeding, and hematoma noted at insertion site

Following a collision while mountain biking, the diagnostic work up of a 22 year-old male has indicated the presence of an acute subdural hematoma. Which of the following pathophysiological processes most likely underlies his diagnosis? - Blood has displaced CSF in the ventricles as a consequence of his coup-contrecoup injury - Blood has accumulated between the man's dura and subarachnoid space - Vessels have burst between the client's skull and his dura - A traumatic lesion in the frontal or temporal lobe has resulted in increased ICP chapter 16

Blood has accumulated between the man's dura and subarachnoid space

Two nursing students are attempting to differentiate between the presentations of immune thrombocytopenic purpura (ITP) and thromobotic thrombocytopenic purprura (TTP). Which of the student's statements best captures an aspect of the two health problems? - ITP can be either inherited or acquire, it involves an enzyme deficiency - Both diseases can result from inadequare peoduction of thrombopoietin by megakaryocytes - TTP can be treated with plasmapheresis, but ITP is best addressed with transfusion of fresh frozen plasma - Both of them involve low platelet counts, but in TTP there can be more, not less hemostasis chapter 22

Both of them involve low platelet counts, but in TTP there can be more, not less hemostasis

A community health nurse is conducting an education session with a group of pregnant women. One of the clients states, "I am not feeling right about getting a flu vaccine while pregnant." What is the nurse's best response? - The influenza vaccine is shown to be safe and will reduce the risk for you getting the flu - Vaccines are a personal choice and you should decide what is right for you and your baby - By getting vaccinated, you may reduce the risk of you and your baby getting influenza - The choice of yours, but none of the substances in the vaccine will harm you or your baby chapter 11

By getting vaccinated, you may reduce the risk of you and your baby getting influenza

When explaining how carbon dioxide combines with water to form carbonic acid as part of an acid-base lecture, the faculty instructor emphasizes that which enzyme is needed as a catalyst for this reaction? - Hydrolases - Carbonic anhydrase - Trypsin - Phenylalanine hydroxylase chapter 8

Carbonic anhydrase

A middle-aged woman is brought to the emergency room after a minor auto accident. Her gait is staggering and unsteady, her speech is slurred and she displays slight nystagmus. The police officer who brought her in says she has not been drinking. Her blood pressure is very high. Which health problem most likely underlies her present state? - Multiple sclerosis - Cerebellar damage caused by cerebrovascular accident - Myasthenia crisis - Guillain-Barre syndrom chapter 15

Cerebellar damage caused by cerebrovascular accident

The nurse will most likely assess which clinical manifestation in a client diagnosed with Creutfeldt-Jakob disease? - Muscle inflammation and edema, making movements very painful - Change in behavior and memory, loss of coordination leading to encephalopathy - Projectile vomiting, hypertension, and drowsiness caused by elevated ICP - Gastrointestinal problems like vomiting and diarrhea chapter 10

Change in behavior and memory, loss of coordination leading to encephalopathy

A 16-year-old female has been brought to her primary care physician by her mother due to the girl's persistent sore throat and malaise. Which fact revealed in the girl's history and examination would lead the physician to rule out infectious mononucleosis? - Blood work reveals an increased white blood cell count - Chest auscultation reveals crackles in her lower lung fields bilaterally - Her liver and spleen are both enlarged - The girl has a temperature of 38.1 C and has enlarge lymph nodes chapter 24

Chest auscultation reveals crackles in her lower lung fields bilaterally

A 4-year-old boy presents with a chronic cough and swollen lymph nodes. His records show that he has been given antibiotics several times in the past year with limited success, most recently for a liver abscess. He also has a recurring fungal condition. Which diagosis is most likely the cause of this child's manifestations? - Ataxia- telangiectasia - Selective IgA deficiency - A deficiency in IgG2 subclass antibodies - Chronic granulomatous disease chapter 12

Chronic granulomatous disease

From the assigned team of clients, which client is at highest risk for the development of endothelial damage that may lead to disseminated intravascular coagulation (DIC)? - Client who had an automobile accident client and sustained three broken ribs resulting in a small pneumothorax - Adolescent client who developed blisters on the back from sunbathing requiring pain medication - Client with chronic obstructive pulmonary disease (COPD) who is diagnosed with pneumonia , most likely from a virus - Client on a monitor displaying frequent premature ventricular contractions (PVCs)

Client with chronic obstructive pulmonary disease (COPD) who is diagnosed with pneumonia , most likely from a virus

Which client, when faced with acute stressful situations, would be considered highest risk for becoming noncompliant with his/her medication regimen? - Teenager whose period is late by 2 weeks and afraid to tell her parents - Client with end-stage renal failure experiencing electrolyte imbalances related to having trouble sticking to the prescribed diet - New mother who is breast-feeding every 2 to 3 hours throughout the day and night - Paraplegic client who forgot his pressure control device and has a small decubitus on the coccyx chapter 7

Client with end-stage renal failure experiencing electrolyte imbalances related to having trouble sticking to the prescribed diet

A nurse on a neurology unit is assessing a female brain-injured client. The client is unresponsive to speech, and her pupils are dilated and do not react to light. She is breathing regularly but her respiratory rate is 45 breaths per minute. In response to a noxious stimulus, her arms and legs extend rigidly. What is her level of impairment? - Vegetative state - Brain death - Coma - Delirium chapter 15

Coma

A nurse is providing care for a 17-year old boy who has experienced recurrent sinus and chest infections throughout his life and presently has enlarged tonsils and lymph nodes. Blood work indicated normal levels of B cells and free immunoglobins but a lack of differentiation into normal plasma cells. The boy is currently is receiving intravenous immunoglobulin (IVIg) therapy. What is the boy's most likely diagnosis? - IgG subclass deficiency - Common variable immunodeficiency - X-linked hypogammaglobulinemia - Transient hypoglobulinemia chapter 12

Common variable immunodeficiency

The nurse is questioning the validity of oxygen saturation readings from the new oximeters on the unit. What action should the nurse take to best determine if the results from oximeter are valid? - Compare the findings with all the clients' results on the unit to determine if there is a trend in the results - Review the literature about this brand of oximeter provided by the company - Repeat the test on the same client to determine if the oximeter offers the same results - Compare clients' results to measurements taken using arterial blood gas analyses chapter 1

Compare clients' results to measurements taken using arterial blood gas analyses

A physician is working with a 30-year-old male client with Down syndrome with Down syndrome who has been admitted with a diagnosis of acute leukemia. Which physical assessment would the physician be more likely to find in an examination of this client than other clients without Down syndrome? - Decreased visual acuity - Congenital heart defects - Hepatomegaly - Diabetes mellitus chapter 5

Congenital heart defects

Several months ago, a 20 year-old male suffered a spinal cord injury brought about by a snowboard trick gone wrong. The lasting effects of his injury include a flaccid bowel and bladder and the inability to obtain an erection. While sensation has been completely preserved in his legs and feet, his motor function is significantly impaired. What type of incomplete spinal cord injury has the man most likely experienced? - Brown-Sequard syndrome - Central cord syndrome - Conus medullaris syndrome - Anterior cord syndrome chapter 15

Conus medullaris syndrome

A nurse in the emergency department admits a male client who has experienced severe frostbite to his hands and toes after becoming lost on a ski hill. The nurse recognizes that which phenomena has contributed to his tissue damage? - Reactive vasodilation has compromised perfusion - Decreased blood viscosity has resulted in interstitial bleeding - Autonomic nervous stimulation has resulted in injury - Decreased blood flow has induced hypoxia chapter 3

Decreased blood flow has induced hypoxia

A 79-year-old female resident of an assisted living facility receives care from a community nurse on a regular basis for a treatment of a chronic venous leg ulcer. Which factor would the nurse be most justified in ruling out as a contributing factor to the client's impaired wound healing? - Decreased fibroblast synthesis - Slow re-epithelialization - A lower skin collagen content than in younger adults - Decreased antibody levels chapter 9

Decreased antibody levels

Your client with end-stage renal disease is receiving 2 units of packed red blood cells for anemia (HgB of 8.2 g/dL [82g/L)). Twenty minutes into the first transfusion, the nurse observes the client has a flushed face, hives over upper body trunk, and is reporting pain in lower back. His vital signs include pulse rate of 110 and BP drop to 95/56. What is the nurse's priority action? - Recheck the type of blood infusing with the chart documentation of client's blood type - Slow the rate of the blood infusion to 50 mL/hour - Document the assessment as the only action - Discontinue the transfusion and begin an infusion of normal saline chapter 23

Discontinue the transfusion and begin an infusion of normal saline

When a Rh-negative mother becomes pregnant by a partner who is Rh-positive, which advanced treatment modality will decrease the risk of the fetus developing hemolytic disease? - Discuss options to prevent any further pregnancy - Discuss an intrauterine transfusion to the affected fetus - Give mother gamma globuline-containing Rh antibody in the first trimester of pregnancy - Perform aminocentesis to draw unconjugated bilirubin and immediate delivery if levels are high chapter 23

Discuss an intrauterine transfusion to the affected fetus

The family of an older adult client is wondering why his "blood counts" are not rising after his last GI bleed. They state, "He has always bounced back after one of these episodes, but this time it isn't happening. Do you know why?" The nurse will respond based on which pathophysiologic principle? - Due to stress, the red blood cells of other older adults are not replaced as promptly as younger people - The doctor may start looking for another cause of his anemia, maybe cancer of the bone - Don't worry about it. We can always give him more blood - Everything slows down when you get older. You just have to wait and see what happens chapter 23

Due to stress, the red blood cells of other older adults are not replaced as promptly as younger people

An 11-year-old-girl is suspected of having Turner syndrome. Which diagnostic test would be most useful component of screening to confirm or rule out the diagnosis? - Echocardiogram - Bone scan - Computed tomography of the head - Liver biospy chapter 5

Echocardiogram

The nurse is caring for a client with jaundice. What evidence will the nurse use to determine if the cause of the jaundice is due to hemolytic condition? - Decreased hemoglobin - Elevated conjugated bilirubin - Liver function tests - Decreased conjugated bilirubin chapter 23

Elevated conjugated bilirubin

The nurse is caring for a client with a diabetic foot ulcer who has difficulty adhering to dietary restrictions. This has resulted in elevated blood glucose levels. What information is most important for the nurse to share with the client? - High blood glucose levels damage your arteries and reduce the blood flow to your wound - Elevations in blood glucose make it more difficult for your white blood cells to fight the infection - We can have the registered dietician come to speak to you about the importance of following your diet - When you do not comply with your diet, it makes it more difficult for us to help you get well chapter 9

Elevations in blood glucose make it more difficult for your white blood cells to fight the infection

A 62-year-old female with a diagnosis of acute and chronic renal failure secondary to diabetes mellitus is receiving her weekly injection of epoetin, a supplementary form of erythropoietin. Which statement best captures the necessity of this medication? - Erythropoietin causes the erythrocyte colony-forming units to proliferate and mature - Erythropoietin is necessary for the accurate sensation of hypoxia that stimulates erythropoiesis - Erythropoietin is needed in order for stem cells to proliferate into committed erythroid precursors - Erythropoietin facilitates the extrusion of the reticulocyte nucleus and the formation of true erythrocytes chapter 23

Erythropoietin causes the erythrocyte colony-forming units to proliferate and mature

A teenage, exposed to West Nile virus a few weeks ago while camping with friends, is admitted with headache, fever, and nuchal rigidity. The teenage is also displaying some lethargy and disorientation. The nurse knows which medical diagnosis listed below may be associated with these clinical manifestations? - Encephalitis - Spinal infection - Lyme disease - Rocky mountain spotted fever chapter 16

Encephalitis

Unbeknownst to her or her care team, a 51 year-old woman's breast cancer has an etiology rooted in the fact that tumor suppressing genes are present but have been silenced. Consequently, she has not synthesized normal cancer-suppressing proteins and neoplasia has resulted. What process has accounted for the woman's cancer? - A DNA repair defect - Chromosomal translocation - Epigenetic mechanisms - The "two-hit" hypothesis of carcinogenesis chapter 6

Epigenetic mechanisms

A nurse is providing care for a client who is immunocompromised following chemotherapy. The nurse knows that which characterization of the adaptive immune system is responsible for the client's disruption in normal immune function? - Effector cells orchestrate the immune response of regulatory cells toward an antigen - Epitopes on antigens are recognized by immunoglobulin receptors following presentation by accessory cells - Accessory cells such as macrophages are engulfed by regulatory cells, stimulating effector cells - Haptens combine to form epitopes that stimulate the response of regulatory and effector cells chapter 11

Epitopes on antigens are recognized by immunoglobulin receptors following presentation by accessory cells

Parents bring their 18-month-old child to the emergency room exhibiting behavior changes and vomiting. They are concerned the child ingested something in the older home they are renovating. Laboratory findings indicate low hemoglobin levels and elevated creatinine. Which diagnostic test should the nurse advocate for first? - Erythrocyte sedimentation rate - Ultrasound of the kidneys - Erythrocyte protoporphyrin (EP) level - Urine test for a mercury level chapter 3

Erythrocyte protoporphyrin (EP) level

Which phenomena best accounts for the increased presence of leukocytes at the site of inflammation? - Existing leukocytes stick to the epithelial cells and move along blood vessel walls - Increased number of leukocytes are released into circulation via cytokine stimulation - Epithelium expresses leukocyte stimulation factors in response to cell injury - Leukocytes are osmotically drawn from circulation into the interstitial space as a result of swelling chapter 9

Existing leukocytes stick to the epithelial cells and move along blood vessel walls

As part of his diagnostic workup, a 77-year-old male is having his ferritin level analyzed. Which explanation by the health care provider regarding the significance and rationale for this test is most accurate? - Ferritin is the activated and useable form of iron that your red blood cells can use to transport oxygen - Ferritin is a stored form of iron that indirectly shows me whether you would benefit from iron pills - Ferritin is a protein-iron complex that allows your red blood cells to make use of the iron that you consume in your diet - Ferritin is the form of iron that is transported in your blood plasma to red blood cells that need it chapter 23

Ferritin is a stored form of iron that indirectly shows me whether you would benefit from iron pills

A client with malignant melanoma has been prescribed alpha interferon, a biologic response modifier. Since this drug prolongs the cell cycle, increasing the percentage of cells in the G0 phase and stimulates NK cells and T-lymphocyte killer cells, the nurse can anticipate that they may experience which common side effects? - Nausea, vomiting, and diarrhea - Opportunistic infections like Candida - Renal damage with increased creatinine level - Fever, chills, and fatigue chapter 6

Fever, chills, and fatigue

During science class, a student asks, "What is the difference between plasma and serum in the blood?" The nurse responds that the primary difference between plasma and serum is that plasma contains: - Hydrogen ions - Heparin - Fibrinogen - White blood cells chapter 21

Fibrinogen

A blood sample is ordered to be collected in a plasma-separator tube, but the nurse can only find serum-separator tubes available. The nurse should be most concerned about the accuracy of which results if the serum-separator tube is used? - Albumin and total protein levels - Fibrinogen and clotting factor levels - Immunoglobulin analysis - Electrolyte measurements chapter 21

Fibrinogen and clotting factor levels

A client at risk for febrile neutropenia related to chemotherapy treatments has been prescribed medication to prevent this reaction. Which medication should the nurse be prepared to administer? - Immunoglobulin M (IgM) antibody - Prednisone, a corticosteroid - Filgrastim, a granulocyte-colony stimulating factor (G-CSF) - Epoetin alfa, an erythropoiesis-stimulating agents (ESAs) chapter 24

Filgrastim, a granulocyte-colony stimulating factor (G-CSF)

The blood work of a 44-year-old male patient with a diagnosis of liver disease secondary to alcohol abuse indicates low levels of albumin. Which phenomena would a clinician be most justified in anticipating? - Impaired immune function - Acid-base imbalances - Fluid imbalances - Impaired thermoregulation chapter 21

Fluid imbalances

A client is brought to the emergency department with reports of shortness of breath. Assessment reveals a full, bounding pulse, severe edema, and audible crackles in lower lungs bilaterally. The nurse notifies the physician to obtain orders for which of these problems? - Hyperkalemia - Fluid volume excess - Hypocalcemia - Hyponatermia chapter 8

Fluid volume excess

The nurse is caring for a client with left-sided heart failure causing chronic activation of the renin-angiotensin-aldosterone system (RAAS). What is the nurse's priority assessment? - Hyperkalemia - Decreased blood pressure when standing - Fluid volume excess - Increased urine output chapter 7

Fluid volume excess

A 14-year old boy, appearing to be intoxicated, arrives in the emergency department where the EMTs report the boy denies consuming anything out of the ordinary, However, an open antifreeze container was found in the boy's room. Which intervention is likely to be prescribed to treat the client's symptoms? - Gastric lavage - Syrup of ipecac - Fomepizole - Sodium bicarbonate chapter 8

Fomepizole

A 63-year-old woman has begun a diet that purports to minimize the quantity and effects of free radicals in her body. Which physiologic process could best underlie her care provider's teaching about her new diet? - Free radicals inhibit the inflammatory response, limiting pre-adaptive response to infection - Free radicals act as direct mediators in the inflammatory process - Free radicals increase cytokine expression and adhesion molecule levels, resulting in increased inflammation - Free radicals contribute to atherosclerosis and decreased immune response chapter 9

Free radicals increase cytokine expression and adhesion molecule levels, resulting in increased inflammation

During a late night study session, a pathophysiology student reaches out to turn the page of her textbook. Which component of her nervous system contains the highest level of control of her arm and hand action? - Cerebellum - Frontal lobe - Basal ganglia - Thalamus chapter 15

Frontal lobe

A client has experienced a pontine stroke which has resulted in severe hemiparesis. What priority assessment should the nurse perform prior to allowing the client eat or drink from the food tray? - Blood pressure reading looking for hypertension - Gag reflex assessing for problems with chewing and swallowing - Cornea blink reflex for ability to see the food tray - Ability to follow commands looking for hearing loss chapter 16

Gag reflex assessing for problems with chewing and swallowing

The nurse volunteering in the medical tent for a road race on a hot humid day is asked to see a runner who has collapsed on the road. The nurse notes he has sunken eyes, a temperature of 100 ' F (37.7'C), and dizziness. These are signs of a fluid volume deficit, Recognizing fluid volume deficit, which of these interventions does the nurse carry out first? - Offer water by mouth - Begin cooling of his body by ice packs - Give him transfusion of FFP - Give him an electrolyte solution by mouth chapter 8

Give him an electrolyte solution by mouth

After several months on a waiting list, a 44-year-old male received a liver transplant 5 days ago. In the last 36 hours, he has developed a rash beginning on his palms and soles, along with abdominal pain and nausea. It has been determined by his care team that the immune response that is causing his symptoms originates not with his own compromised immune components but with those introduced with his new organ. This man's most likely medical diagnosis is: - Acute transplant rejection - Graft-versus-host disease (GVHD) - Hyperacute organ rejection - T-cell mediated graft rejection chapter 12

Graft-versus-host disease (GVHD)

Which glycoprotein is responsible for treating such diseases as bone marrow failure following chemotherapy and hematopoietic neoplasms such as leukemia? - Growth factors and cytokines - T lymphocytes and natural killer cells - Natural killer cells and granulocytes - Neutrophils and eosinophils

Growth factors and cytokines

During a period of stress, the nurse asks the client to close his eyes and think of a calm, relaxing place where he can feel the wind blowing on his cheek and small the salty air from the ocean. This is an example of utilizing which type treatment for stress reduction? - Yoga - Hypnosis - Guided imagery - Biofeedback chapter 7

Guided imagery

A client diagnosed with H. pylori asks the nurse, "How canan infection occur in the stomach since it is an acid environment?" The nurse responds: - H. Pyloria is a virus and is still being researched as to how it is able to survive in the stomach acids - This parasite secretes an enzyme called coagulase, which protects the pathogen from the antibodies - We have many infectious agents that can live in an acidic environment with a pH more than 8.0 - H. pylori produces an enzyme called urease that converts gastric juices into ammonia, which neutralizes acidic stomach environment chapter 10

H. pylori produces an enzyme called urease that converts gastric juices into ammonia, which neutralizes acidic stomach environment

A 19 year-old intravenous drug user was exposed to the HIV virus 3 weeks ago and is experiencing a rapid proliferation in viral load. Which statement best captures an aspect of the process of HIV replication that underlies this proliferation? - Free HIV RNA is able to attach to the cell coat of CD4+ cells - Expression of reverse transcriptase by CD4+ cells allows replication of HIV cells rather than new lymphocytes - HIV is able to change its RNA into DNA to allow for replication by CD4+ cells - The cytoplasm of CD4+ cells provides a protected environment for the replication of RNA by HIV chapter 12

HIV is able to change its RNA into DNA to allow for replication by CD4+ cells

While teaching about HIV/AIDS to a group of high school seniors, the school health nurse will begin by explaining the basic facts. Which information will this likely include? - HIV is different from other viruses since it is a retrovirus that selectively attacks the body's immune cells - There are two types of HIV, but that is endemic to the U.S. is HIV type 2 - Like all viruses, HIV is a genetic material made from DNA with long molecules that carry genetic information - HIV type 1 for some reason rarely develops into full-blown AIDS chapter 12

HIV is different from other viruses since it is a retrovirus that selectively attacks the body's immune cells

A client with Parkinson disease has challenged himself to maintain mobility for longer than the physician predicts. He strives every day to walk 5 to 10 steps father than the day before. This phenomenon researched by social psychologists, is known as: - Positivism - Assertiveness - Stubbornness - Hardiness chapter 7

Hardiness

A 9 year-old boy has been brought to the emergency department by his father who is concerned by his son's recent fever, stiff neck, pain, and nausea. Examination reveals a petechial rash. Which assessment question by the emergency room physician is most appropriate? - Was your son born with any problems that affect his bone marrow or blood? - Is your son currently taking any medications? - Has your son had any sinus or ear infections in the last little while? - Does your son have a history of cancer? chapter 16

Has your son had any sinus or ear infections in the last little while?

The unique clinical presentation of a 3 month-old infant in the emergency department leads the care team to suspect botulism. Which assessment question posed to the parents is likely to be most useful in the differential diagnosis? - Is there any mold in your home that you know of? - Has your baby ever been directly exposed to any chemical cleaning products? - Have you ever given your child any honey or honey-containing products? - Is there any family history of neuromuscular diseases? chapter 15

Have you ever given your child any honey or honey-containing products?

A nurse is interviewing a client with fever, myalgia, headache, and lethargy. Which question is most important for the nurse to ask related to identifying the cause of these symptoms and the possible need for quarantining the client? - Are these symptoms similar to an illness you have had in the past? - Did you receive an influenza vaccine in this season? - Have you or any family members traveled outside the country recently? - Have you been exposed to anyone who has been vaccinated recently? chapter 10

Have you or any family members traveled outside the country recently?

A 48-year-old mean who has been HIV positive for 6 years has just learned that he has been diagnosed with Kaposi sarcoma (KS). Which fact accurately conveys an aspect of his diagnosis? - An opportunistic Epstein-Barr virus underlies the man's KS - He is likely to have lesions on his skin, mouth, or GI tract - Intense pain was probably his first manifestations of KS - Heterosexual contact most likely underlies his HIV and subsequent KS chapter 12

He is likely to have lesions on his skin, mouth, or GI tract

A 26-year-old man who survived childhood acute lymphocytic leukemia (ALL), one of the most common childhood cancers, now complains of weakness, fatigue, and shortness of breath. His treatment for ALL likely included anthracyclines. What is the most likely cause of his symptoms? - Recurrent of ALL - Heart failure resulting from childhood chemotherapy - Hormonal dysfunction resulting from childhood chemotherapy - CNS problems resulting from childhood therapy chapter 6

Heart failure resulting from childhood chemotherapy

Which infectious agent would be a site-specific pathogen and not spread throughout the body via the circulatory system? A client diagnosed with: - Salmonella typhi related to ingestion of contaminated food or water - Helicobacter pylori diagnosed after an endoscopic procedure - N. meningitidis infection of child in daycare setting - B. burgdorferi caused by a tick bite chapter 10

Helicobacter pylori diagnosed after an endoscopic procedure

A student states, "It seems like helper T cells do a lot more than just 'help' the cellular immunity process". Which of the following responses listed below best conveys an aspect of the role of CD4+ helper T cells in immunity? - Helper T cells are key to the hematopoiesis that produces all the components of the immune system - Without helper T cells, no antigens would be presented - Without helper T cells, the wrong antibodies would end up being produced - Helper T cells play a major role in stimulating and regulating the whole process chapter 1`1

Helper T cells play a major role in stimulating and regulating the whole process

A 39-year-old female with HIV has been characterized as a typical progressor by her care team, and is experiencing an increase in her manifestations and complaints as her CD4+ count declines. Which health problem would her care team most likely attribute to a cause other than her HIV? - Her recent diagnosis of bacterial pneumonia - Her increasing confusion and disorientation - Her esophagitis that has been linked to herpes simplex infection - Her decreased bone density and recent fractures chapter 12

Her decreased bone density and recent fractures

Which assessment and laboratory findings would be most closely associated with acute leukemia? - Increased blood urea nitrogen and bone pain - High blast cell counts and fever - Decreased oxygen partial pressure and weight loss - Increased serum potassium and sodium levels

High blast cell counts and fever

A college student has been experiencing frequent headaches that he describes as throbbing; he is reporting difficulty concentrating while studying. Upon cerebral angiography. he is found to have an arteriovenous malformation. Which pathophysiologic concept is likely responsible for his symptoms? - Localized ischemia with areas of necrosis noted on CT angiography - Increased tissue perfusion at the site of the malformation - Hydrocephalus and protein in the cerebrospinal fluid - High pressure and local hemorrhage of the venous system chapter 16

High pressure and local hemorrhage of the venous system

A 60-year-old male officer worker presents to a clinic complaining of new-onset of lower back pain that has been worsening over the last 6 weeks. The nurse knows that which component of his physical assessment and history is most indicative of a serious pathologic process (like aortic aneurysm or cancer)? - When supine, passive rising of his leg to 90 degrees results in hamstring pain - He has needed regular nonsteroidal anti-inflammatory drugs to control the pain in recent weeks - His onset of pain has been gradual and he has no prior history of lower back problems - His pain is relieved by extended bed rest chapter 15

His onset of pain has been gradual and he has no prior history of lower back problems

A deficiency in which of these would result in an inhibition of the inflammatory response? - Vitamin K - B cells - Histamine - Helper T Cells chapter 9

Histamine

When explaining to parents what is occurring when their child has an acute bronchial asthma attack, the nurse will emphasize that which mediator is primarily responsible for the bronchial constriction? - Tree pollen - T-lymphocyte proliferation - Histamine - Mold dust chapter 12

Histamine

When explaining the final stages of the inflammatory response to pathogens, the nurse should include which educational topics? = Margination, which is a process whereby white cells (leukocytes) stick to the endothelium and accumulate along the blood vessel - How the body can kill the pathogen by generating toxic oxygen and nitrogen products, producing such things as nitric oxide and hydrogen peroxide - The increase in vascular permeability, which lets fluids leak into the extravascular tissues - The promotion of tissue regeneration, whereby monocytes and macrophages produce potent prostaglandines and leukotrienes chapter 9

How the body can kill the pathogen by generating toxic oxygen and nitrogen products, producing such things as nitric oxide and hydrogen peroxide

While undergoing a kidney transplant from a non-family member, the client's transplanted kidney has just had the arterial clamps removed. The OR staff notices that the organ is turning purple with no urine output. When explaining to the family why they had to remove the donor kidney, the nurse will anticipate that the surgeon would most likely include which statement? - Previous exposure to the HLA antigens is responsible for the high titers of complement-fixing antibodies that cause the rejection - The circulating B and T lymphocytes are just doing their job - Hyperacute rejection occurs because antibodies against HLA antigens are deposited in vessels, causing necrosis - Obviously, there has been a mismatch during the human leukocyte antigens (HLA) testing chapter 12

Hyperacute rejection occurs because antibodies against HLA antigens are deposited in vessels, causing necrosis

A male client has an autosomal dominant disorder. The client and his partner are considering starting a family. Which statement indicates the client has an adequate understanding of the genetic basis of this health problem? - My children who don't have the disease still run the risk of passing it on to their children - I know there's no way of accurately determining the chance that my child will inherit the disease - I know that new genetic mutations won't occur between generations - I know that a single mutant allele is to blame for the health problem Chapter 5

I know that a single mutant allele is to blame for the health problem

A nurse working in a busy orthopedic clinic is asked to perform the Tinel sign on a client having problems in her hand/wrist. In order to test Tinel sign, the nurse should give the client which direction? - I'm going to tap this tuning fork, place it on the side of your thumb, then tell me what you are feeling in your hand and wrist - I'm going to tap (percuss) over the median nerve in your wrist, tell me what sensation you feel while I am doing this. Does the sensations stay in the wrist or go anywhere else? - Stand tall, arms at your side, shut your eyes; place your tip of your index finger to your nose - Hold your wrist in complete flexion, keep it in this position for 60 seconds. How does your hand feel after placing it in a neutral position? chapter 15

I'm going to tap (percuss) over the median nerve in your wrist, tell me what sensation you feel while I am doing this. Does the sensations stay in the wrist or go anywhere else?

A client arrives at the emergency department reporting muscle cramps and spasms in the legs and feet; tingling in the fingers, toes, and lips; and an anxiety. Serum calcium level is 4.8 mg/dL (1.2 mmol/L). Suspected tetany, which medication should the nurse be prepared to administer? - vitamin D - magnesium sulfate IV - IV calcium gluconate - a thiazide diuretic chapter 8

IV calcium gluconate

While taking a daily walk, the nurse is asked by a neighbor what centric fusion (Robertsonian) translocation means, The neighbor tells the nurse that a family member has been diagnosed with this and is not afraid to have children. Given this diagnosis, what may be the potential risks for her offspring? - Since the extremely short fragment only contains a small amount of genetic material, there should be no additional risk than the normal population - Cleft lip with cleft palate is frequently associated with this translocation of genetic material - If chromosome 21 is involved, there is a high risk for producing a child with Down syndrome - This translocation of genetic material places the child at high risk for having multiple limb abnormalities chapter 5

If chromosome 21 is involved, there is a high risk for producing a child with Down syndrome

A community health nurse is conducting the nutritional component of a class for new mothers. Which teaching point would be most justified? - Be aware that cow's milk depletes your baby's supply or iron - Iron supplementation is not necessary provided you are breast-feeding your infant - Your infant needs the same amount of iron as you, but has far fewer sources for obtaining it - If you choose to feed your baby with formula, ensure that it is iron-fortified chapter 23

If you choose to feed your baby with formula, ensure that it is iron-fortified

The nurse is caring for an 8-month-old male client who has just been diagnosed with hemophilia A, The parents ask about the likelihood that any future children would be born with the condition. What is the nurse's best response? - Since you have already had a son with hemophilia, the chance that you will have another child with the condition is unlikely - I cannot offer an opinion as the pattern of inheritance is unknown and you need to seek genetic counseling - There is a 50% chance that your next child will have hemophilia, but the severity varies and it could be a mild form - If you have another son, there is a 50% chance that he will also have hemophilia chapter 5

If you have another son, there is a 50% chance that he will also have hemophilia

The parent of a child with allergies has been told the child will have a blood test done for immunoglobulin measurement. The nurse should tell the parent that which immunoglobulin will be tested? - IgE - IgG - IgA - IgM chapter 11

IgE

Three days ago, a mother delivered her full-term infant who had been identified as having an in utero infection. The infant is receiving antibiotic and phototherapy, and the mother is breastfeeding. Which of the following types of immunoglobulins could most reasonably be expected to predominate in the infant's immune system? - IgG, IgA, IgM - IgA, IgM, IgD - IgE, IgG, IgD - IgM, IgD, Igm chapter 11

IgG, IgA, IgM

While being on subcutaneous heparin injections for deep vein thrombosis during her latter pregnancy, a patient begins to experience major side effects. Her OB physician has called in a specialist who thinks the patient is experiencing heparin-induced thrombocytopenia. The nurse should anticipate which of the following orders? - Order to decrease the dose of heparin from 5000 units bid to 3000 units bid - Immediately discontinue the heparin pathway - Infusion of FFP stat - A switch to warfarin 2.5 mg once/day chapter 22

Immediately discontinue the heparin pathway

A 51-year-old female has been found to have metastatic lesions in her lung but her oncologist is unsure of the primary tumor site. Which procedure is most likely to aid in this determination? - Tissue biopsy - Microarray technology - Tumor markers - Immunohistochemistry chapter 6

Immunohistochemistry

The daughter of a client with metastatic cancer is confused as to why the client is receiving radiation therapy and asks the nurse, "Why are we still treating the cancer? The plan was only for comfort care." What is the nurse's best response? - This type of radiation has no serious adverse effects so we often apply it in palliative cases as there is no harm - Radiotherapy is slowing tumor growth so you can have more time with your family member, but is not curative - In this case, the radiation is being used to help alleviate the bone pain your family member is experiencing - You will need to discuss the plan of care with your family member as I am not at liberty to discuss is with you chapter 6

In this case, the radiation is being used to help alleviate the bone pain your family member is experiencing

When educating a client with a wound that is not healing, the nurse should stress which dietary modifications to ward off some of the negative manifestations that can occur with inflammation? - This is the one time whereby you should eat more fat (both polysaturated and saturated) so you can absorb more fat-soluble vitamins - Since there is a loss of plasma proteins, you should increase your intake of organ meats like liver - Increase the amount of calcium in your diet, especially drinking milk and eating cheese - Increase your intake of oily fish and fish oil so that you will increase your absorption of omega-3 polyunsaturated fatty acids chapter 9

Increase your intake of oily fish and fish oil so that you will increase your absorption of omega-3 polyunsaturated fatty acids

A client with poorly controlled diabetes mellitus presents to the emergency department with suspected ketoacidosis. Which diagnostic results would be most likely to confirm the diagnosis? - Increased anion gap, base deficit - Decreased anion gap, decrease urine ammonium level - Low O2 levels, increased anion gap, base excess - High ammonia levels, decreased anion gap, high potassium chapter 8

Increased anion gap, base deficit

A client who has been awaiting the results of a bone marrow biopsy for several days is experiencing stress as a result of uncertainty and the possibility that abnormal cell growth may be detected. A physical examination and blood work would most likely yield which result? - Increased respiratory rate; increased levels of testosterone - Increased gastrointestinal motility; decreased thyroid-stimulating hormone - Increased blood pressure and heart rate; increasing antidiuretic hormone (ADH) - Pupil dilation; increased somatomedin C chapter 7

Increased blood pressure and heart rate; increasing antidiuretic hormone (ADH)

Health care team members know recently an increased incidence of infections such as West Nile virus and SARS do not match previously established patterns. Which phenomenon constitutes the most significant contributor to the spread of new diseases in the United States? - Decreased living standard and public health standards in urban areas - Increased ease and speed of travel for individuals and groups - Genetic variation and mutation by microorganisms - Drug resistance by bacterial and protozoal infections chapter 10

Increased ease and speed of travel for individuals and groups

Which aspect of a client's site of inflammation would help the care provider rule out chronic inflammation? - Infiltration of lymphocytes - Proliferation of fibroblasts - Increased neutrophils - High levels of macrophages chapter 9

Increased neutrophils

Following a course of measles, a 5-year-old girl developed scattered bruising over numerous body surfaces and was diagnosed with immune thrombocytopenic purpura (ITP). As part of her diagnostic workup, blood work was performed. Which of the following results is most likely to be considered unexpected by the health care team? - Increased thrombopoietin levels - Decreased platelet count - Normal vitamin K levels - Normal leukocyte levels chapter 22

Increased thrombopoietin levels

The geriatrician providing care for a 74-year-old man with a diagnosis of Parkinson disease has recently changed the client's medication regimen. What is the most likely focus of the pharmacologic treatment of the man's health problem? - Preventing demyelization of the efferent cerebellar pathways - Maximizing acetycholine release from synaptic vesicles at neuromuscular junctions - Increasing the functional ability of the underactive dopaminergic system - Preventin axonal degradation of motor neurons chapter 15

Increasing the functional ability of the underactive dopaminergic system

A nurse practitioner is providing care for a client with low levels of the plasma protein gamma globulin. The nurse would recognize that the client is at risk of developing which health problem? - Anemia - Blood clots - Jaundice - Infections chapter 21

Infections

A 23-year-old man has received a recent diagnosis of appendicitis. The nurse providing care for the man is explaining that the inflammation of his appendix is playing a role in his body's fight against the underlying infectious process. Which teaching points should the nurse eliminate from client education? - Inflammation can help to remove the body tissue cells that have been damaged by infection - Inflammation helps your body to produce the right antibodies to fight the infection - Inflammation will start your body on the path to growing new, healthy tissue at the site of infection - Inflammation ultimately aids in eliminating the initial cause of the cell injury to your appendix chapter 9

Inflammation helps your body to produce the right antibodies to fight the infection

A school-age child with a history of asthma has brought a note home from school stating there has been one case of meningitis (Neisseria meningitidis) in the school. Since the mother is a nurse, she is very concerned since she knows the portal of entry of this pathogen is: - through a cut or abrasion that may occur on the playground - Ingestion such as when children share their drink with their friends - Inhalation via the respiratory tract such as through breathing or yawning - direct contact with contaminated object such as a pencil chapter 10

Inhalation via the respiratory tract such as through breathing or yawning

A 40-year-old woman who experiences severe seasonal allergies has been referred by her family physician to an allergist for weekly allergy injections. The woman is confused as to why repeated exposure to substances that set off her allergies would ultimately benefit her. Which phenomenon best captures the rationale for allergy desensitization therapy? - Repeated exposure stimulates adrenal production of epinephrine, mitigating the allergic response - Injections of allergens simulate production of IgG, combining with the antigens to prevent activation of IgE antibodies - Allergens in large, regular quantities overwhelm the IgE antibodies that mediate the allergic response - Repeated exposure to offending allergens binds the basophils and mast cells that mediate the allergic response chapter 12

Injections of allergens simulate production of IgG, combining with the antigens to prevent activation of IgE antibodies

A 44-year-old hospitalized client with a diagnosis of end-stage acquired immunodeficiency syndrome (AIDS) has been placed on neutropenic precautions that limit his interaction with visitors, staff, and other clients. What is the underlying for these precautions? - His antibody-mediated immunity is compromised by his low production of neutrophils - Neutropenia limits the ability of his CD4+ helper cells to present antigens - Cyclic neutropenia limits his body's ability to fight various infections - Insufficient levels of neutrophils make him particularly susceptible to infections chapter 24

Insufficient levels of neutrophils make him particularly susceptible to infections

A 61-year-old client with a 40 pack-year history of cigarette smoking and chronic obstructive pulmonary disease (COPD) is experiencing an increase in arterial levels of CO2. This change stimulates the inspiratory center in the medulla oblongota, which in turn causes the diaphragm to contract more forcefully and increase respiratory rate. Which term best describes the role of the inspiratory center? - Sensor - Integrator - Feedback system - Effector chapter 7

Integrator

The nurse should anticipate that she will need to teach the client newly diagnosed with multiple sclerosis how to give injections if which mediation to assist with reducing exacerbations is prescribed? - Mitoxantrone - Plasmapheresis - Interferon beta - Corticosteroids chapter 15

Interferon beta

A 60-year-old male client with an acute viral infection is receiving interferon therapy, The nurse is teaching the client about the ways interferon differs from other anti-infective therapies. What point should the nurse include? - Interferon will suppress apoptosis to prevent the death of healthy cells - Interferon helps limit macrophage activity to reduce the reaction to the infection - Interferon stimulates the activity of natural killer cells that attack viruses - Interferon will only target the cells affected by the virus and not affect other cells chapter 11

Interferon stimulates the activity of natural killer cells that attack viruses

The nurse is caring for a client who has been on an oral corticosteroid for several years for an autoimmune condition. The nurse notes several bruises of different stages of resolution on the client. What is the nurse's best response to this finding? - Obtain a STAT order for repeat platelet count and international normalized ratio (INR) - Complete a thorough pain assessment related to the bruising - Request an order for vitamin K to prevent life-threatening hemorrhage - Interview the client regarding side effects of corticosteroid therapy chapter 22

Interview the client regarding side effects of corticosteroid therapy

The nurse is collecting a blood sample for an ordered CBC. The required tube contains EDTA so the nurse will: - Place the tube immediately on ice to preserve the action of the EDTA on the sample - Invert the tube several times to promote clotting so serum can be fully separated - Avoid inverting the tube to prevent hemolysis, which could affect the accuracy of the results - Invert the tube several times to prevent the blood from coagulating chapter 22

Invert the tube several times to prevent the blood from coagulating

Misinterpreting her physician's instructions, a 69-year-old woman with a history of peripheral artery disease has been taking two 325 mg tablets of aspiring daily. How has this most likely affected her hemostatic status? - The binding of an antibody to platelet factor 4 produces immune complexes - The client's prostaglandin TXA2 levels are abnormally high - She is at risk of developing secondary immune thrombocytopenic purpura (ITP) - Irreversible aceytlation of platelet cyclooxygenase activity has occurred chapter 22

Irreversible aceytlation of platelet cyclooxygenase activity has occurred

A single mother who was raised in a traumatic environment is very protective of her child. She expresses a desire to keep her child from experiencing stressful situations in hopes of promoting healthy development. What response by the nurse is most helpful? - You are creating more stress for yourself while protecting your child and this is not healthy - I think it is admirable that you are protecting your child from negative life experiences - It is normal to experience stressors periodically and this can enhance healthy development - You can never prevent all stressful situations: they are a reality of growing into adulthood chapter 7

It is normal to experience stressors periodically and this can enhance healthy development

As part of her prenatal education, a 29 year-old woman who is pregnant with her first child is receiving teaching from her primary care provider. Which of the following statements by the woman reflects an accurate understanding of HIV transmission? - I know it's possible but it's comforting that the chances of my child contracting my HIV are actually very low - It's discouraging to know that my breast milk can pass on HIV to my baby - I know my baby is safe from HIB while in the womb, but the delivery will place him or her at real risk - I'm relieved to learn that a caesarean delivery will protect my baby from being born HIV positive chapter 12

It's discouraging to know that my breast milk can pass on HIV to my baby

A nurse is changing the wound dressing on a coccyx-region pressure injury of an immobilized client. The existing dressing is saturated with both watery, clear discharge and foul, grayish-colored liquid. Which entry in the client's chart most accurately documents these findings? - Purulent discharge and fibrinous exudates noted on existing dressing - Abscess activity noted to coccyx wound - Plasma proteins and membranous exudates present on existing dressing - Large amounts of suppurative and serous exudates noted chapter 9

Large amounts of suppurative and serous exudates noted

A student is attempting to trace the feedback cycle involved in the stress response. Which neural structure is thought to be the central integrating site for the stress response? - Cerebral cortex - Hypothalamus - Locus caeruleus - Reticular formation chapter 7

Locus caeruleus

A person who has been diagnosed with HIV infection 12 years ago and still has a CD4+ cell count of 800 cells/uL and a low viral load is considered to be in which clinical group? - Delayed progressor - Rapid progressor - Long-term nonprogressor - Typical progressor chapter 12

Long-term nonprogressor

A 16-year-old girl has broken her arm while snowboarding and is shocked at the amount of swelling at the injury site. Which statement best explains the physiologic rationale for her swelling? - Potent vasodilation increases the total volume of vascular space at the site of inflammation - Migration and proliferation of mast cells, neutrophils, and platelets to the injury site occupy an increased volume of tissue - Osmotic flow of plasma into the intravascular space causes increased blood volume and interstitial fluid - Loss of plasma protein causes an increase in interstitial osmotic pressure chapter 9

Loss of plasma protein causes an increase in interstitial osmotic pressure

A client with a diagnosis of end-stage liver failure has arterial blood gas results indicated altered pH. Which of these consequences of liver failure has most likely caused the acid-base disturbances? - Low albumin and plasma globulin levels - Renal excretion of HCO3 in the presence of excess base - Transcompartmental exchange of H and potassium ions - Conservation of formation of new HCO3 by the kidneys chapter 8

Low albumin and plasma globulin levels

The nurse knows which component is needed for long-lasting immunity in a client with a diagnosis of sepsis without the causative agent identified? - Neutrophils - Colony-stimulating factors - Lymphocytes - Natural killer cells chapter 11

Lymphocytes

A 53-year-old female hospital has received a kidney transplant following renal failure secondary to hypertension. The teaching prior to transplant made her aware that she would need to take anti-rejection drugs for the rest of her. life. Which aspect of the immune system underlies this necessity? - Anti-rejection drugs will stimulate the production of familiar MHC molecules - The lack of identifiable major histocompatibility complex (MHC) molecules will stimulate the innate immune response - Donor organ antibodies will be identified as foreign and stimulate an immune response - MHC molecules will never develop in the cells of the donor organ and effector cells will be continually stimulated chapter 11

MHC molecules will never develop in the cells of the donor organ and effector cells will be continually stimulated

Which teaching point would be most appropriate with a client who has a recent diagnosis of Von Willebrand disease? - It's important that you avoid trauma - Make sure that you avoid taking aspirin - Clotting factor VIII can help your body compensate for the difficulty in clotting - Your disease affects your platelet function rather than clot formation chapter 22

Make sure that you avoid taking aspirin

During a discussion with a client recently diagnosed with stage IV kidney cancer, which statement is most accurate? - You likely had some kidney pain that you associated with back strain instead of cancer - The health care community needs to do a better job informing people that blood clots in the urine is not normal - Many times, kidney cancer goes completely undetected until a metastatic lesion is found in the lung - Most people ignore the blood they see in their urine thinking they have a kidney stone chapter 6

Many times, kidney cancer goes completely undetected until a metastatic lesion is found in the lung

A client presents with sudden, violent diarrhea and vomiting after consuming chicken and potato salad 8 hours prior at the beach on a hot day. The nurse knows which statement best matches the phase of the infectious process of food poisoning? - Sudden incubation of active replication - Maximum impact of infectious process - Subacute prodromal phase - Insidious prodromal phase chapter 10

Maximum impact of infectious process

Which diagnostic finding is likely to result in the most serious brain insult? - Increased ICP accompanied by hyperventilation - Moderate decrease in brain tissue volume secondary to a brain tumor removal - High intracellular concentration of glutamate - Mean arterial pressure (MAP) that equals intracranial pressure (ICP) chapter 16

Mean arterial pressure (MAP) that equals intracranial pressure (ICP)

A client's arterial blood gases reveal normal oxygen level, pH 7.50 PCO2 level of 50 mmHg (6.65 kPA) and HCO3 level of 30 (30 mmol/l). The client's respiratory rate is 12 breaths/min and all other vital signs are within normal range. What is this client's most likely diagnosis? - Metabolic acidosis - Respiratory acidosis - Respiratory alkalosis - Metabolic alkalosis chapter 8

Metabolic alkalosis

An adult female, newly diagnosed with possible leukemia, arrives for a bone marrow biopsy. The admission laboratory work includes red blood cell count (RBC): 3.4x10^6/uL (3.4 x10^12/L); white blood cell count (WBC): 2,500/uL (2.5 x10^9/L); platelet count of 80 x 10^3/uL (80 x10^9L); all below normal values. What assessment is the highest priority for this client? - Assessing heart sounds for any new murmurs - Monitoring for any type of bleeding - Checking breath sounds twice a day for signs of pneumonia - Forcing fluids to maintain renal function chapter 21

Monitoring for any type of bleeding

A pregnant client is having premature labor cramps and is diagnosed with preeclampia. The admitting prescriptions include starting an IV infusion of magnesium sulfate. What intervention will the nurse provide this client? - Evaluating the client's ECG for prolongation of QT interval - Placing the client on telemetry and monitoring for prolonged PR interval and wide QRS complex - Assessing for osteoporosis and bone fractures - Monitoring for changes in neuromuscular status like lethargy, confusion, and hyporeflexia chapter 8

Monitoring for changes in neuromuscular status like lethargy, confusion, and hyporeflexia

A 30-year-old male's blood work and biopsies indicate that he has proliferating osteoclasts that are producing large amounts of IgG. What is the man's most likely diagnosis? - Acute myelogenous leukemia - Hodgkin lymphoma - Acute lymphocytic leukemia - Multiple myeloma chapter 24

Multiple myeloma

The nurse in the emergency department knows clients exposed to Clostridium botulinum, an agent of bioterrorism, would likely be exhibiting which clinical manifestation listed below? - Blindness and respiratory distress - Hemorrhage from all orifices resulting in signs of shocks and coma - Frothy, odiferous diarrhea and dehydration - Muscle weakness in extremities eventually leading to paralysis of respiratory muscles chapter 10

Muscle weakness in extremities eventually leading to paralysis of respiratory muscles

A client with a diagnosis of schizophrenia has been admitted with suspected hyponatremia after consuming copious quantities of tap water. Which finding does the nurse anticipate uncovering related to this problem? - High urine specific gravity, tachycardia and a weak thready pulse - Low blood pressure, dry mouth, and increased urine osmolality - Increased hematocrit and blood urea nitrogen; seizures - Muscle weakness, lethargy, and headaches chapter 8

Muscle weakness, lethargy, and headaches

The nurse is caring for a client with a longstanding diagnosis of hypocalcemia secondary to kidney disease. The nurse observes which clinical manifestations in this client? - Loss of appetite and reports of nausea - Muscular spasms and reports of tingling in hands/feet - Lethargy and change in level of consciousness - High fluid intake and copious amounts of dilute urine output chapter 8

Muscular spasms and reports of tingling in hands/feet

The physician knows the client, G1P0 has correctly understood the prenatal education regarding sexually transmitted infection as evidenced by which of the following statements listed below? - I know that my baby will need observation for HIV signs and symptoms in the weeks following my delivery - My baby could become infected either across the placenta or during the birth itself - Out of the various infectious agents, gonorrhea and chlamydia pose the greatest risks of transmissions from mother to child - If I receive prophylactic immunization, I will reduce my baby's chance of being born with an illness chapter 10

My baby could become infected either across the placenta or during the birth itself

An 81-year-old male client who has a diagnosis of orthostatic hypotension is experiencing an episode of particularly low blood pressure. The man's body has responded by increasing levels of angiotensin II in the bloodstream, a hormone which decreases glomerular filtration rate in the kidneys and contributes to an increase in blood pressure. Which phenomena best describes what has occurred? - Positive feedback - Adaptation - Negative feedback - Homeostasis chapter 7

Negative feedback

A client presented to the emergency department of the hospital with a swollen, reddened, painful leg wound and has been diagnosed with methicillin-resistant Staphylococcus aureus (MRSA) cellulitis. The client's physician has ordered a complete blood count and white cell differential. Which blood component would the physician most likely anticipate to be elevated? - Basophils - Neutrophils - Platelets - Eosinophils chapter 9

Neutrophils

Following an injury resulting in a small cut from a knife, the first cells to go to the area of the cut would be the: - Albumin - Neutrophils - Erythrocytes - Basophils chapter 21

Neutrophils

A student arrives at the health clinic anxious and afraid. The student found in an enlarged lymph node in the groin area that is extremely painful to touch and "knows" it is cancer. What information should the health care provider relay to this student about lymphadenitis? - Increased sexual activity in college is a precursor to development of genital cancers - The student is correct. A warming sign of cancer is a lump or bump that should not be there - Not all swollen lymph nodes are due to cancer. It could be caused by an infection in genital region - Swollen lymph nodes that are painful to touch are most likely characteristics of neoplasms chapter 9

Not all swollen lymph nodes are due to cancer. It could be caused by an infection in genital region

The school nurse has several children with hemophilia A. After recess, one hemophilia student comes to the school nurse complaining of pain in the knee from falling on the playground. The nurse notes there is swelling in the knee and pain on palpation. The nurse should: - Notify parents to pick up the child and possibly administer Factor VIII - Wrap the knee in an ace bandage for compression - Administer some NSAIDs to relieve the pain - Apply some warm compresses to the knee chapter 22

Notify parents to pick up the child and possibly administer Factor VIII

The nurse at a long-term care facility encourages the older adults to drink even though they may not feel thirsty at the time. Which statement supports the nurse's action? - The metabolic needs for both fluid and sodium in older adults differ from those of younger individuals - The renin-angiotensin-aldosterone system (RAAS) is less able to facilitate sodium clearance in older adults - Older adults often experiences a decrease in the sensation of thirst, even when serum sodium levels are high - Regulation and maintenance of effective circulating volume by the kidneys is less effective in older adults chapter 8

Older adults often experiences a decrease in the sensation of thirst, even when serum sodium levels are high

The nurse is caring for a client who is suffering from high levels of chronic stress. On what should the nurse focus as a negative effect on the client's health due to chronically elevated levels of cortisol? - Blood glucose levels are decreased due to excessive energy expenditure - The immune system is bolstered in response to an impending threat - Osteoblast activity and protein synthesis are suppressed to refocus energy - Attention, arousal, and respiratory rate are increased to prepare for a response chapter 7

Osteoblast activity and protein synthesis are suppressed to refocus energy

An 8-week-old boy has been recently diagnosed with a severe combined immunodeficiency (SCID). His parents have performed a significant amount of research on the Internet and have brought a large amount of material to discuss with their care provider. Which of the following statements best reflects an accurate understanding of their son's health situation? - The antibodies that our son produces are mismatched to the infections that he was born with, and encounters - Our son likely has a deficiency of B-lymphocytes and can't produce antibodies - We read that gene therapy could cure our son; we'd like you to look into that option - We feel guilty, because dietary and environmental factors have been shown to contribute to SCID chapter 12

Our son likely has a deficiency of B-lymphocytes and can't produce antibodies

Amniocentesis has suggested that a couple's first child will be born with sickle cell disease. The parents are unfamiliar with the health problem and their caregiver is explaining the complexities. Which statement by the parents would suggest a need for further teaching or clarification? - Not all of his or her red cells will be sickled, but low oxygen levels can cause them to become so - Sickled cells can block his or her blood vessels especially in the abdomen, chest, and bones - Our baby's red cells are prone to early destruction because of their weak membranes - Our son or daughter likely won't show the effects of sickling until he or she is school-ages because of the different hemoglobin in babies chapter 23

Our son or daughter likely won't show the effects of sickling until he or she is school-ages because of the different hemoglobin in babies

After being thrown off the back of a bull, the bull rider can move their arms but has loss of motor function in the lumbar and sacral segments of the spinal cord. This is usually referred to as being a: - Quadriplegia - Tetraplegia - Paraplegia - Anterior cord syndrome chapter 15

Paraplegia

An 80-year-old woman is slated for total hip replacement the following day, and is experiencing a large amount of stress around her potential surgical outcomes. Which is most likely to be uninvolved in the physiologic response to her stress? - Hypothalamus - Parathyroid - Adrenal cortex - Pituitary gland chapter 7

Parathyroid

A client with hypercalcemia and hypercalciuria has just passed a kidney stone. The nurse recognizes which of these laboratory studies should also be assessed? - Bone scan - Serum magnesium level - Plasma phosphate levels - Parathyroid hormone level chapter 8

Parathyroid hormone level

A client who has a diagnosis of an autoimmune disease asks his nurse why is it that their immune system does not attack all of the cells that make up his body. Which of the following aspects of pathogen recognition in the innate immune system listed below would underlie the nurse's response? - Leukocytes possess pathogen associated molecular pattern (PAMPs) - Intraepithelial lymphocytes and natural killer cells possess specific, highly diverse receptors - Pattern recognition receptors (PRRs) ensure cells are correctly identified - Normal host cells excrete inhibitory proteins that are detected by natural killer cells chapter 11

Pattern recognition receptors (PRRs) ensure cells are correctly identified

When discussing the sequence of clot dissolution, the science instructor will talk about which item that begins the process> - Platelets - Dabigatran - Plasminogen - A2 Plasmin inhibitor chapter 22

Plasminogen

A 71-year-old male client with a hsitory of myocardial infarction (MI) and peripheral vascular disease (PVD) has been advised by his family physician to begin taking 81 mg aspirin once daily. which statement best captures an aspect of the underlying rationale for the physician's suggestion? - Aspiring inhibits the conversion of fibrinogen into fibrin and consequent platelet plug formation - Platelet aggregation can be precluded through inhibition of prostaglandin production by aspirin - Aspirin helps to inhibit ADP action and minimizes platelet plug formation - Aspirin can reduce unwanted platelet adhesion by inhibiting TXA2 synthesis chapter 21

Platelet aggregation can be precluded through inhibition of prostaglandin production by aspirin

Which client is most likely to have impairments to the wound healing process? A client with: - Chronic obstructive pulmonary disease - Poorly controlled blood sugars with small blood vessel disease - A diagnosis of multiple sclerosis and consequent impaired mobility - Congenital heart defects and anemia chapter 9

Poorly controlled blood sugars with small blood vessel disease

A 5-year-old needs to undergo a bone marrow biopsy. When educating the parents, which site should the nurse state is the most commonly used? - Sternum - Anterior iliac crest - Posterior iliac crest - Spinous chapter 21

Posterior iliac crest

The family of a 68 year-old man who is in the end stages of small cell lung cancer are distraught at his visible body wasting that has worsened in recent weeks. Which of the following phenomena best accounts for the client's anorexia and cachexia? - Inadequate cellular metabolism of glucose results from tumor results - Inadequate food intake due to symptoms and treatment results in loss of blood muscle and fat - High-fat losses coupled with preservation of muscle mass exaggerate the appearance of wasting - Products of the tumor itself as well as hypermetabolic state cause cachexia chapter 6

Products of the tumor itself as well as hypermetabolic state cause cachexia

Which statement best captures an aspect of the process of hematopoiesis? - Various subtypes of pluripotent stem cells eventually differentiate into the cellular components of blood - Progenitor cells differentiate into precursor cells - Self-replicating precursor cells differentiate into specific CSFs - Colony-stimulating factors (CSFs) produce cytokines that activate progenitor cells chapter 21

Progenitor cells differentiate into precursor cells

A 29-year-old construction worker got a sliver under his fingernail 4 days ago, The affected finger is now reddened, painful, swollen, and warm to the touch. Which of the following hematologic processes is most likely occurring in the bone marrow in response to the infection? - Phagocytosis by myelocytes - Increased segmented neutrophil production - Proliferation of immature neutrophils - High circulatory levels of myeloblasts chapter 21

Proliferation of immature neutrophils

A client is asked to stand with feet together, eyes open, and hands by the sides. Then the client is asked to close the eyes while the nurse observes for a full minute. What assessment is the nurse performing? - Crossed-extensor reflex - Segmental reflex - Posture - Proprioception chapter 15

Proprioception

A client has been suffering from fatigue and the health care provider suspects anemia. Which test result will the health care provider consult when investigating the cause of the anemia? - Hemoglobin - Hematocrit - Red blood cell (RBC) - RBC indices chapter 21

RBC indices

A client with a history of epilepsy is brought to the emergency department due to seizure activity. What information about the client is most important for the nurse to collect? - Allergies to medications - Recent medication history - Family history of seizure disorders - Type of seizure activity experienced chapter 16

Recent medication history

A client has been inhaling viruses periodically while on a cross-country flight. Which situation listed below would most likely result in the stimulation of the client's T lymphocytes and adaptive immune system? - Presentation of a foreign antigen by a familiar immunoglobulin - Cytokine stimulation of a T lymphocyte with macrophage or dendritic cell mediation - Recognition of a foreign major histocompatibility complex (MHC) molecule - Recognition of a foreign peptide bound to a self-major histocompatibility complex (MHC) molecule chapter 11

Recognition of a foreign peptide bound to a self-major histocompatibility complex (MHC) molecule

A client has been diagnosed with a brain tumor that cannot be removed surgically. During each office visit the nurse will be assessing the client for syndrome of inappropriate antidiuretic hormone (SIADH). Which assessment would alert the clinic nurse that the client may be developing this complication? - Muscle cramps, tingling around the mouth/lips, unexplained bruising - Fever, diarrhea, nausea - Report of decreased urine output, no edema noted in ankles, increasing headache - Hyperglycemia, dry mucous membranes, projectile vomiting chapter 8

Report of decreased urine output, no edema noted in ankles, increasing headache

In the neurotrauma unit, a teenager with a closed head injury related to an automobile accident is experiencing high intracranial pressure (ICP). He is intubated and on a ventilator. One treatment for this is to allow him to progress into which acid-base imbalance in an attempt to lower ICP? - Metabolic Alkalosis - Respiratory acidosis - Respiratory alkalosis - Metabolic acidosis chapter 8

Respiratory alkalosis

If an Rh-negative mother is giving birth to an Rh-positive infant, the nurse should be prepared to administer: - antihistamines like dyphenhydramine - Rh immune globulin - a monoclonal antibody like infliximab - alpha interferon chapter 23

Rh immune globulin

Which breakfast diet contains chemicals that likely contribute to a higher rate of cancer risk? - Pancakes made with buttermilk and warm maple syrup - English muffin with jelly and scrambled eggs - Sausage fried in reused grease and biscuits and gravy - Tofu stir-fried with peppers and onions topped with cheddar cheese chapter 6

Sausage fried in reused grease and biscuits and gravy

A female dental assistant has developed signs and symptoms of a latex sensitivity, and is undergoing allergy testing as well as blood work. Which component of the assistant's blood work would most likely be the focus of her health care provider's analysis? - Serum IgE immunoassays - Analysis of class II MHC antigens - Serum CD8+ levels - Serum B-lymphocytes levels

Serum IgE immunoassays

The nurse caring for a lung cancer client with metastasis to the brain suspects the client has developed a paraneoplastic syndrome known as Syndrome of Inappropriate Antidiuretic Hormone (SIADH) secretion. Which laboratory results in this client who has gained 3 lb in a day would alert the nurse to the possibility of SIADH? - BUN of 8 mg/dL (2.86 mmol/L) - Serum potassium of 5.0 mEq/L (5.0 mmol/L) - Serum sodium of 115 mEq/L (115 mmol/L) - Hematocrit of 40% chapter 6

Serum sodium of 115 mEq/L (115 mmol/L)

A 77-year-old female hospital client has contracted Clostridium difficile during her stay and is experiencing severe diarrhea. Which statement best conveys a risk that this woman faces? - She is prone to isotonic fluid volume excess - She could develop third-spacing edema as a result of plasma protein losses - She is at risk of compensatory fluid volume overload secondary to gastrointestinal water and electrolyte losses - She is susceptible to isotonic fluid volume deficit chapter 8

She is susceptible to isotonic fluid volume deficit

A 66-year-old female client has presented to the emergency department because of several months of intermittently bloody stool which has recently become worse. The woman has since been diagnosed with a gastrointestinal bleed secondary to overuse of nonsteroidal anti-inflammatory drugs that she takes for her arthritis. The health care team would realize that which situation is most likely? - The client will be at risk for cardiovascular collapse or shock - She will have delayed reitculocyte release - She will have iron-deficiency anemia due to depletion of iron stores - The woman has depleted blood volume due to her ongoing blood loss chapter 23

She will have iron-deficiency anemia due to depletion of iron stores

The nurse is performing discharge teaching for a client who has recovered from sepsis following an open cholecystectomy. What should the nurse's primary focus? - Follow-up appointments with health care providers - Signs and symptoms of infection - Exercises that are permitted postoperatively - Dietary changes needed after cholecystectomy chapter 10

Signs and symptoms of infection

A 30-year-old woman who has given birth 12 hours prior to displaying signs and symptoms of disseminated intravascular coagulation (DIC). The client's husband is confused as to why a disease of coagulation can result in bleeding. Which statement by the nurse best characterizes DIC? - Massive clotting causes irritation, friction, and bleeding in the small blood vessels - These same hormones and bacteria that cause clotting also causes bleeding - Excessive activation of clotting causes an overload of vital organs, resulting in bleeding - So much clotting takes places that there are no available clotting components left, and bleeding ensues chapter 22

So much clotting takes places that there are no available clotting components left, and bleeding ensues

Which statement in an accurate descriptor of the role of viruses in human infections? - Viruses have no genetic material of their own - Some viruses are capable of transforming normal host cells into malignant cells - Viruses require stimulation after a latent period before they are able to produce symptoms - Viruses are often implicated in cases of transmissible neurodegenerative disease chapter 10

Some viruses are capable of transforming normal host cells into malignant cells

A new client who suffered a myocardial infarction requires angioplasty and stent placement. he has arrived to his first cardiac rehabilitation appointment. in this first session, a review of the pathogenesis of coronary artery disease is addressed. which statement by the client verifies to the nurse that he has understood the nurse's teachings about coronary artery disease? - "If you do not exercise regularly to get your heart rate up, blood pools in the veins causing a clot which stops blood flow to the muscle and you have a heart attack" - "Sounds like this began because of inflammation inside my artery that made it easy to form fatty streaks which lead to my clogged artery" -All I have to do is stop smoking and then I won't have any more heart attacks - My artery was clogged by fat so I will need to stop eating fatty foods like french fries every day chapter 1

Sounds like this began because of inflammation inside my artery that made it easy to form fatty streaks which lead to my clogged artery

A client has been identified as having an excess of macrophage inhibitory factor, causing the client to have inhibited movement and activity of macrophages. Which process listed below would the health care team member expect to remain unaffected? - Initiation of adaptive immunity - Amplification of the immune response - Destruction of virus-infected or tumor cells - Specificity and memory of the immune response chapter 11

Specificity and memory of the immune response

A child has been diagnosed with thalessemia. Which of the following comorbidities may occur as a result of having thalessemia? - Splenomegaly - Iron deficiency - Neutropenia - Hypocoagulation chapter 23

Splenomegaly

A hospital client is receiving intravenous infusion of heparin for treatment of a pulmonary embolus. Which phenomenon is most likely occurring, resulting in the drug's therapeutic effect? - Inhibition of ADP-induced platelet aggregation - Deactivation of the intrinsic clotting pathway - Suppression of fibrin formation - Inhibition of vitamin K synthesis in the liver chapter 22

Suppression of fibrin formation

A geriatrician is following a number of clients on a subacute geriatric medical unit, some of whom require diagnostic blood work. Which client would be most likely to have an erythrocyte sedimentation rate (ESR) screening test ordered? An adult with: - Orthostatic hypertension and syncopal episodes - Systemic lupus erythematosus - Congestive Heart failure - A diagnosis of Alzheimer disease and depression chapter 21

Systemic lupus erythematosus

The nurse knows that the cells primarily programmed to remove the invading organisms and remember the antigen to respond rapidly during the next exposure are: - CD4 and CD8 cells - White blood cells and platelets - Natural Killer (NK) cells and macrophages - T-lymphocytes and B-lymphocytes chapter 11

T-lymphocytes and B-lymphocytes

A nurse is at a long-term care facility provides care for an 85 year-old man who has had recent transient ischemic attacks (TIAs). Which statement best identifies future complications associated with TIAs? - TIAs are an accumulation of small deficits that may eventually equal the effects of a full CVA - TIAs resolve rapidly but may place the client at an increased risk for stroke - TIAs are a relatively benign sign that necessitate monitoring but not treatment - TIAs are caused by small bleeds that can be a warning sign of an impending stroke chapter 16

TIAs resolve rapidly but may place the client at an increased risk for stroke

A 12-year-old client who experienced trauma is not having trouble dealing with many forms of conflict in the home and becomes distraught. What is the best approach for the nurse to recommend to the parents? - Avoid having disagreements in the home until the child is well enough to come with the discord - Place the child in a quiet space from any disruptions when there are disagreements - Talk about the past trauma often and encourage the child to think positively about the future - Request a mild sedative that the parents can administer to the child if becoming distraught chapter 7

Talk about the past trauma often and encourage the child to think positively about the future

A couple who are pregnant with their first child have made an appointment with a clinical geneticist to discuss prenatal screening, The man states that they "just want to make sure that there is nothing wrong with our baby." How could the clinician best respond to this statement? - Testing the umbilical blood and performing amniocentesis can give us some information, but not a guarantee - We can't rule out all abnormalities, but a routine fetal tissue biopsy can yield useful information - You need to be aware that if abnormalities are detected, termination is normally required - Prenatal screening is not usually necessary unless you are among a high-risk group chapter 5

Testing the umbilical blood and performing amniocentesis can give us some information, but not a guarantee

A client overheard the provider discussing the case and mentioning a "mutation int eh TP53 gene." The client asks the nurse, "What does that mean?" Which response is the most appropriate for this client? - The TP53 gene was likely damaged when you were in utero and exposed to a toxin - This means that they are looking for a monoclonal antibody that will selectively kill your tumor - The TP53 gene is a group of tumor suppressor genes associated with lung, breast, and colon cancer - All cancers result from an abnormality in some gene or chromosome chapter6

The TP53 gene is a group of tumor suppressor genes associated with lung, breast, and colon cancer

An occupational health nurse working with police officers wishes to apply Selye's general adaptation syndrome theory in practice. Which concept should the nurse apply? - Period of stress are developmentally negative for the person experiencing them - The nature of a stress response is determined by the objective severity of the stressor - The alarm stage involves the release of cortisol and catecholamines - Systemic illnesses can sometimes result from the resistance stage of stress response chapter 7

The alarm stage involves the release of cortisol and catecholamines

A hospital laboratory technician performing routine blood analysis as part of an inpatient's assessment is examining the sample in a test tube following processing in a centrifuge and the addition of an anticoagulant. Which observation would the technician interpret as an anomaly? - The middle layer of cells appears white to grey in color - The bottom layer of blood in the tube accounts for around 40% to 45% of the total volume - The top layer of cells is too thin to visualize without microscopy - The yellowish fluid on the top of the sample appears to constitute around one-quarter of the total volume - chapter 21

The bottom layer of blood in the tube accounts for around 40% to 45% of the total volume

A 6-year-old boy with intellectual disability secondary to fragile X syndrome has been admitted to the hospital with a mitral valve prolapse. The health care worker caring for the child should have which concepts as part of her knowledge base regarding fragile X syndrome? - The boy's mother had a 100% chance of transmitting the defective gene to her son. - Genes of the boy's Y chromosome can be affected in addition to the X chromosome - An affected mother who carries one normal and one mutant allele has a 75% chance of transmitting the gene to her daughters - The boy will pass the gene to all his future daughters who will become carriers chapter 5

The boy will pass the gene to all his future daughters who will become carriers

A client asks the health care provider why his lower legs look purple. The health care provider will base the response on which pathophysiologic principle? - The bruising around the ankles is due to the fact that it is dependent area where the capillary pressure is higher - Morbid obesity causes veins to enlarge and bleed into tissues due to stress the abdomen is placing on the vascular system - There is a problem with the plasminogen levels - Too much trauma breaks capillaries and they bleed into the tissue chapter 22

The bruising around the ankles is due to the fact that it is dependent area where the capillary pressure is higher

A male client with a history of heavy alcohol use has been admitted to the hospital for malnutrition and suspected pancreatitis. The client's diagnostic workup suggests alcoholic ketoacidosis as a component of his current health problems. He is somewhat familiar with the effect that drinking has had on his nutrition and pancreas, but is wholly unfamiliar with the significance of acid-base balance. How best could his care provider explain the concept to him? - The multitude of chemical reactions that take place in your body depend on your body fluids being slightly acidic - Your body is highly dependent on what food and fluid you consume to keep itself at a functioning level of slight non-acidity - The chemical processes that take place throughout your body are thrown off very easily when your body is too acidic or not acidic enough - The healthy function of your kidneys and your lungs require a specific level of pH in your body chapter 8

The chemical processes that take place throughout your body are thrown off very easily when your body is too acidic or not acidic enough

A 2-year-old girl has had repeated ear and upper respiratory infections since she was born. A pediatrician has determined a diagnosis of transient hypogammaglobulinemia of infancy. What is the physiologic origin of the child's recurrent infections? - The child lacks the antigen presenting cells integral to normal B cell antibody production - The child's immune system is unable to synthesize adequate immunoglobulin on its own - The child had a congenital absence of IgG antibodies, which her body is only slowly beginning to produce independently - The child was born with IgA and IgM antibodies, suggesting intrauterine infection chapter 12

The child's immune system is unable to synthesize adequate immunoglobulin on its own

A clinician is conducting an assessment of a male client suspected of having a disorder of motor function. Which of the following assessment findings would suggest a possible upper motor neuron (UMN) lesion? - The client's muscles appear atrophied - The client displays increased muscle tone - The client has decreased deep tendon reflexes - The client displays weakness in the distal portion of his limbs chapter 15

The client displays increased muscle tone

A counseling psychologist is working with a 30-year-old female client who is experiencing the symptoms of posttraumatic stress disorder (PTSD) following a house fire several months prior. Which of the client's diagnostic results could most likely be interpreted as a manifestation of PTSD? - The client has increased levels of growth hormones - The client has decreased levels of angiotensin II - The client has decreased levels of cortisol The client has decreased levels of norepinepherine chapter 7

The client has decreased levels of cortisol

A 56 year-old female hospital client with history of alcohol abuse is receiving intravenous (IV) phosphate replacement. Which health problem will this IV therapy most likely resolve? - The client has an irregular heart rate and a thready pulse - The client has an accumulation of fluid in her peritoneal cavity - The client has abdominal spasms and hyperactive reflexes - The client is acidotic and has impaired platelet function chapter 8

The client is acidotic and has impaired platelet function

When explaining to a client admitted for stress-induced supraventricular tachycardia, the nurse will incorporate which statement about what happens in the body as a result of excess stress? - The primary role of the parasympathetic nervous system is to stimulate the vagus nerve - The corticotrophin-releasing factor stimulates the release of norepinephrine, which is responsible for "fight-or-flight" reaction to stress - Endorphins are released from the brain every time we experience stress - The pituitary gland is ultimately responsible for growth, metabolism, and maturation, all of which are important when the body is stressed chapter 7

The corticotrophin-releasing factor stimulates the release of norepinephrine, which is responsible for "fight-or-flight" reaction to stress

An electrician who has been working 14 to 16 hour days for several weeks in order to ensure the financial survival of his business presents to his family physician with complaints of persistent headaches and insomnia. His family physician attributes the physical complaints to the ongoing stress likely causing by which component of the stress response? - The fact that his company is heavily in debt - The fact that he takes diuretic medication for his hypertension - The fact that he has previously had difficulty coping with stress - The fact that he is not physically active on a regular chapter 7

The fact that he has previously had difficulty coping with stress

The clinical educator of a hospital medical unit has the mandate of establishing evidence-based practice guidelines for the nursing care on the unit. Which statement most accurately captures a guiding principle of the nurse's task? - Guidelines are synonymous with systematic research reviews - The need for continuity and standardization of guidelines will mean that they will be fixed rather than changeable - The guidelines will combine individual expertise with external systematic evidence - Evidence - based guidelines will be rooted in research rather than nurses' subjective practice preferences and experiences

The guidelines will combine individual expertise with external systematic evidence

A client is suspected of having an infection, but the microorganism suspected as the cause cannot be cultured. The client asks how the health care provider will identify the organism. What is the nurse's best response as to the methods used? - The health care provider will introduce cultured, marked antibodies to your blood sample and observe for a reaction with antigens - The health care provider will release purified antigens into your circulation to observe whether you produce the relevant antibodies - The health care provider will administer empiric antimicrobials, and observe your response to treatment - The health care provider will observe for a cytopathic effect on biopsy tissue samples from the your mucosa chapter 10

The health care provider will introduce cultured, marked antibodies to your blood sample and observe for a reaction with antigens

A client with a diagnosis of hemolytic anemia has gone to a community-based laboratory for follow-up blood work. The lab technician confirms with the client that hematocrit is one of the components of the blood work. The client replies, "I thought the point of the blood work was to see how many red blood cells I have today." How could the technician best respond to the client's statement? - The hematocrit measures the mass that your red blood cells account for in a quantity of your blood - The result will indicate how many of your red blood cells are new and young. and will indicate your body's production rate of red cells - Your hematocrit measures the average size of your red blood cells and indirectly measures your oxygen-carrying capacity - This result will tell your care provider about the number of red blood cells in a given quantity of your blood plasma chapter 23

The hematocrit measures the mass that your red blood cells account for in a quantity of your blood

The nurse knows which statement below is appropriate to be included in an education session for a 21-year-old male with a diagnosis of malaria? - Your infection likely began with the introduction of a fertilized protozoal ova from a mosquito - The protozoa responsible have hijacked genetic material of your cells in order to reproduce - The infectious organisms are considered tiny, single-celled animals, given their complete eukaryotic machinery - You are very tired because the pathogens are utilizing the ATP that your own cells need chapter 10

The infectious organisms are considered tiny, single-celled animals, given their complete eukaryotic machinery

The nurse is caring for a 30-year-old woman who is diagnosed with iron deficiency anemia. The client is confused how this condition developed because she ate many foods high in iron. How should the nurse respond? - The iron deficiency anemia was most likely the result of chronic blood loss over a period of time - Iron deficiency anemia is actually more likely linked to deficits of vitamin B12 and folate in the diet - Some people have a condition that impairs the ability to use the iron they take in effectively - Diet deficiencies are the most common reason, so it is likely the intake was not sufficient chapter 23

The iron deficiency anemia was most likely the result of chronic blood loss over a period of time

A 77-year-old client diagnosed with chronic obstructive pulmonary disease (COPD) is experiencing impaired gas exchange and CO2 retention, despite a rapid respiratory rate. Which pathophysiologic principle would the health care team expect if the client's compensatory mechanisms are working? - Arterial blood gas sampling indicates a pH in the range of 7.45-7.55 - The kidneys are likely to reabsorb H+ and secrete HCO3 - The body will produce excess metabolic CO2 - The kidneys will adapt with an increase in plasma HCO3 and pH will increase chapter 8

The kidneys will adapt with an increase in plasma HCO3 and pH will increase

A 40-year-old male client is shocked to receive a diagnosis of mature B-cell lymphoma, and is doing research on his diagnosis on the internet. Which statement that he reads on various websites is most reliable? - The lymph nodes are usually affected, and often the spleen and bone marrow - Doctors are able to diagnose mature B-cell lymphoma by the presence of Reed-Sternberg cells - Unlike many other lymphomas, mature B-cell lymphoma is often self-limiting and treatment is focused on symptoms - Like most forms of Hodgekin lymphoma, mature B-cells lymphoma often requires radiation treatment chapter 24

The lymph nodes are usually affected, and often the spleen and bone marrow

A 60-year-old woman is suspected of having non-Hodgkin lymphoma (NHL). Which aspect of her condition would help to rule out Hodgkin lymphoma? - Her neoplasm originates in secondary lymphoid structures - The presence of Reed-sternberg cells has been confirmed - The lymph nodes involved are located in a large number of locations in the lymphatic system - The woman complains of recent debilitating fatigue chapter 24

The lymph nodes involved are located in a large number of locations in the lymphatic system

A female client with a new diagnosis of systemic lupus erythematosus (SLE) has been told that this is an autoimmune disease whereby the immune system is attacking the body's cells and tissue. She knows that she has inflammation and tissue damage. She asks her nurse to explain "What cells in the body are triggering this inflammation?" The nurse response: - Mainly the adrenocorticoids like cortisone are responsible for all your inflammation" - Mineralocorticoids like aldosterone usually begin the process of inflammation - The lymphocytes that migrate to the brain where they secrete cytokines, which trigger inflammation - Primarily, this acute stress reaction is associated with stimulation of the autonomic nervous system, which causes stiffness in the joints chapter 7

The lymphocytes that migrate to the brain where they secrete cytokines, which trigger inflammation

Which physiologic process would be considered a positive feedback mechanism? - Increased production of white blood cells (WBC) in response to a microorganism - Shivering in response to low environmental temperature - The release of anti-diuretic (ADH) from the posterior pituitary gland - The platelet-aggregation mechanism activating clot formation chapter 7

The platelet-aggregation mechanism activating clot formation

A nurse is conducting an immunization clinic when a 14-year-old client asks, "Does this vaccine make me immune to the disease?" What is the nurse's best reponse? - The vaccine imitates a disease without causing illness, resulting in the creation of antibodies to the disease - The vaccine will not make you immune but will improve your chances of surviving an infection - The vaccine contains medicines that are designed to combat the microorganisms that cause this disease - The vaccine introduces substances that replicate and can kill the microorganism if you are exposed chapter 11

The vaccine imitates a disease without causing illness, resulting in the creation of antibodies to the disease

An oncology nurse who has worked for many years providing care for children with cancer has taken a job on an adult oncology unit of a hospital. What differences might the nurse anticipate in this new job? - More clients will be receiving treatment for leukemia - Thur nurse will be working with more clients who has blastomas - A greater proportion of the clients will have cancer that involves the hematopoietic system - There will be a greater number of cancers that are epithelial in origin chapter 6

There will be a greater number of cancers that are epithelial in origin

A potential donor is angry at the personal nature of the questions about HIV risk factors that he is required to answer at a blood collection center, and states that simple blood testing should suffice. How can the nurse at the center best respond? - Even though blood tests are completely accurate, the high stakes of blood donation and transfusion mean that double measures are appropriate - There are some very uncommon subtypes of the HIB virus that are not detectable by current testing methods - There's a period shortly after someone is infected with HIB when blood tests might still be negative - There's a chance that persons who are asymptomatic but HIV positive can have their antibodies missed by serum testing chapter 12

There's a period shortly after someone is infected with HIB when blood tests might still be negative

After assessment, the nurse asks how long the client has had red, pinpoint hemorrhages on the lower legs. The client responds, "This is the first time I have noticed this. What is wrong with me that is causing these small hemorrhages?" Which response by the nurse is most accurate? - These hemorrhages are called petechiae and occur when platelets are deficit - Your platelets are developing a deformity as they are being produced by the bone marrow - Your might have pooling of all your platelets in the spleen. I will assess for that next - More than likely, you bumped something and these are a result of trauma to the vessel chapter 22

These hemorrhages are called petechiae and occur when platelets are deficit

A client has been diagnosed with anemia. The physician suspects an immune hemolytic anemia and orders a Coombs test. The client asks the nurse what this test will tell the doctor. The nurse replies: - They will look at your RBCs under a microscope to see if they have an irregular shape (poikilocytosis) - They will wash your RBCs, then mix the cells with a reagent to see if they clump together - They are looking for the presence of antibody or complement on the surface to the RBC - They will be looking to see if you have enough ferritin in your blood chapter 23

They are looking for the presence of antibody or complement on the surface to the RBC

Which statement by a client who has a new diagnosis of non-Hodgekin lymphoma (NHL) demonstrates a sound understanding of the diagnosis and treatment of the health problem? - They confirmed my diagnosis with a lymph node biopsy and I'll get radiation treatment soon because it's a fairly early stage - Since the tests show NHL, I'm going to pursue my options for palliative care because I'm committed to dying with dignity - They took a sample of my lymph nodes and I'll be having surgery soon that will hopefully cure my lymphoma chapter 24

They confirmed my diagnosis with a lymph node biopsy and I'll get radiation treatment soon because it's a fairly early stage

Which statement best conveys a characteristic of red blood cells? - They help maintain the body's fluid balance - They are self-replicating - They contribute to the maintenance of blood pH - They lack organelles and soluble enzymes chapter 21

They contribute to the maintenance of blood pH

The nurse is preparing to administer rasburicase to a client undergoing chemotherapy. How should the nurse explain the purpose of this medication to the client? - Rasburicase is a unique antiemetic that will reduce your nausea - This drug helps reduce uric acid levels and protect your kidneys - This medication protects your bladder from the toxic effects of chemotherapy - Chemotherapy causes high phosphate and this medication reduces this chapter 24

This drug helps reduce uric acid levels and protect your kidneys

A toddler is displaying signs/symptoms of weakness and muscle atrophy. The pediatric neurologist suspects it may be a lower motor neuron disease called spinal muscular atrophy (SMA). The client's family asks how he got this. The nurse will respond: - This is a segmental demyelination disorder that affects all nerve roots and eventually all muscle groups as well - This is a degenerative disorder that tends to be inherited as an autosomal recessive trait - This could result from playing in soil and then ingesting bacteria that is now attaching his motor neurons - No one really knows how this disease is formed. We just know that in time, he may grow out of it chapter 15

This is a degenerative disorder that tends to be inherited as an autosomal recessive trait

The nurse is caring for a client who has just been diagnosed with shingles. The client states, "I have not been around anyone with this condition. How could I have caught it?" What is the nurse's best response? - The virus is the same one that causes cold sores and is manifesting in a different way this time, so it is very common - Shingles is caused by recent exposure to someone who has active herpes zoster virus or has just been vaccinated - This is a latent virus that you likely were exposed to as a child and it has not become active in the form of shingles - This virus has a long prodromal stage so you could have been exposed over a month ago and not been aware chapter 10

This is a latent virus that you likely were exposed to as a child and it has not become active in the form of shingles

A child with leukemia hears the health care provider tell the parents that the child has "pancytopenia." This child asks the nurse, "What does pancy" mean? Am I going to die?" Which response by the nurse is most appropriate for this 10-year-old child? - This means the chemotherapy is attacking your bone marrow and your body has to find a way to fight back. No, you are not going to die - This means the chemotherapy has lowered your blood count, including red blood cells, white blood cells, and platelets. You are very sick, but we are working hard to make you healthy again." - This is very serious and could possibly lead to death if you start bleeding badly - This is just a way of saying that you are sick and need to rest more. Your medicine is fighting your cancer cells so that you will not die chapter 22

This means the chemotherapy has lowered your blood count, including red blood cells, white blood cells, and platelets. You are very sick, but we are working hard to make you healthy again."

The family member of a client with terminal metastatic cancer who is experiencing cachexia-related weight loss asks why the nurse why the client is losing weight despite taking in a large amount of calories per day. What is the nurse's best response? - We can consult with the doctor and dietician about tube feeding to prevent more weight loss - We likely need to increase the calories more; you are welcome to bring food from home - This weight loss is related to the cancer itself and occurs despite an intake of adequate calories - When clients are at this stage of metastatic cancer the food being eaten is not being digested chapter 6

This weight loss is related to the cancer itself and occurs despite an intake of adequate calories

Following a motor vehicle accident 3 months prior, a 20-year-old female who has been in a coma since her accident has now had her condition declared a persistent vegetative state. How can her care providers most accurately explain an aspect of her situation to her parents? - Your daughter has lost all cognitive functions as well as all her basic reflexes - Your daughter's condition condition is an unfortunate combination with total loss of consciousness but continuation of all other normal brain function - Though she still goes through a cycle of sleeping and waking, her condition is unlikely to change - If you or the care team notices any spontaneous eye opening, then we will change our treatment plan chapter 16

Though she still goes through a cycle of sleeping and waking, her condition is unlikely to change

A 53-year-old man presents with inability to concentrate., itching in his fingers and toes, elevated blood pressure, and unexplained weight loss. He is diagnosed with primary polycythemia. What will be the primary goal of his treatment? - To increase the amount of oxygen distributed by his red bed blood cells - To reduce the viscosity of his blood - To control his hypertension - To reduce the mean size of his red cells chapter 23

To reduce the viscosity of his blood

New parents are upset their 2-day old infant is requiring phototherapy for hyperbilirubinemia. Which factors would the pediatrician be most likely to rule out as a contributor to high bilirubin levels? - Hepatic immaturity of the infant - The fact that the infant is being breast-fed - Hypoxia - Transitioning of hemoglobin F (HbF) to hemoglobin A (HbA) chapter 23

Transitioning of hemoglobin F (HbF) to hemoglobin A (HbA)

A 1-year-old child who has experienced low platelet counts and bacterial susceptibility has been admitted to a pediatric medical unit of a hospital for treatment of Wiskott-Aldrich syndrome. The nurse who has admitted the child to the unit would anticipate which short-term and longer-term treatment plans? - Transfusion of clotting factors XII and XIII and serum albumin; splenectomy - Neutropenic precautions; fresh frozen plasma transfusions; treatment of gastrointestinal symptoms - Treatment of eczema; management of bleeding; bone marrow transplant - Intravenous immunoglobulin (IVIg) treatment; thyroidectomy chapter 12

Treatment of eczema; management of bleeding; bone marrow transplant

A client has been diagnosed with herpes simplex virus. The client states that, " modern medicine produces more and more antivirals every year and so the treatment should be simple." Which statement is the best response? - The cell coat of viruses is particularly resilient to be available synthetic antivirals - The recent rise of drug resistance has significantly hampered the elimination of viruses - The use of antivirals is severely limited by unwanted adverse effects that they cause - Treatment options for viruses are often limited because what destroys viruses often damages your own body cells chapter 10

Treatment options for viruses are often limited because what destroys viruses often damages your own body cells

A 60-year-old man has presented to a clinic and is requesting screening for tumor markers after reading about them in a magazine. What can the clinician most accurately tell the man about the clinical use of tumor markers? - Tests for the presence of tumor markers are limited by the fact that they are only accurate in the very early stages of cancer - Tumor markers are a very useful screening tool, but they only exist for a very few types of cancer - Tumor markers are an excellent screening tool, but it's only practical to test for those cancers that you're at risk of - Tumor markers alone aren't enough to confirm whether you have cancer or not, so they're not a very useful screening tool - chapter 6

Tumor markers alone aren't enough to confirm whether you have cancer or not, so they're not a very useful screening tool

A 70 year-old male has been diagnosed with a stroke that resulted infarct to his cerebellum. Which clinical finding would be most closely associated with cerebellar insult? - Unsteady gait and difficulty speaking and swallowing - Tremor, rigidity, and bradykinesia - Difficulty staring movements, stopping movement, and maintaining rhythmic movements - Flaccid loss of muscle tone chapter 15

Unsteady gait and difficulty speaking and swallowing

Which process would most likely be considered an anomaly during the cellular phase of inflammation? - Macrophage activity - Vasoconstriction - Platelet aggregation - Migration of phagocytic white cells chapter 9

Vasoconstriction

A client in the intensive care unit who has just been a brain tumor has experienced a sharp decline . The care team suspects that water and protein have crossed the blood-brain barrier and been transferred from the vascular space into the client's interstitial space. Which diagnosis best captures this pathophysiology? - Vasogenic edema - Focal hypoxia - Cytoxic edema - Hydrocephalus chapter 16

Vasogenic edema

A 14-year-old boy has been diagnosed with infectious mononucleosis. Which pathophysiologic phenomenon is most responsible for his symptoms? - EBV inhibits the maturation of white cells within his peripheral lymph nodes - The Epstein-Barr virus (EBV) is lysing many of the boy's neutrophils - Viruses are killing some of his B cells and becoming incorporated into the genome of others - The virus responsible for mononucleosis inhibits the maturation of myeloblasts into promyelocytes chapter 24

Viruses are killing some of his B cells and becoming incorporated into the genome of others

A 44-year-old female client presents to the emergency department with abnormal bleeding and abdominal pain that is later attribute to gallbladder disease. Which diagnosis would the medical team be most justified in suspecting as a cause of the client's bleeding? - Hemophilia B - Vitamin K deficiency - Idiopathic immune thrombocytopenic purpura (ITP) - Excess calcium chapter 22

Vitamin K deficiency

A 36-year-old woman with a diagnosis of antiphospholipid syndrome is receiving a scheduled checkup from her nurse practitioner. Which teaching point would the nurse most likely prioritize? - We need to ensure your birth control pills don't contain estrogen - You'll need to avoid taking nonsteroidal anti-inflammatory drugs when you have menstrual cramps - It's important for you to do regular physical activity and maintain a healthy body weight - Good nutrition and blood sugar control are important in your case chapter 22

We need to ensure your birth control pills don't contain estrogen

Which procedure reduces the potential for infection primarily by addressing the portal of entry? - Disposing of soiled clothing and bed linens in a dedicated receptacle - Wiping down common areas with buffered bleach on a regular basis - Isolating clients who have antibiotic-resistant infections - Wearing gloves when contact with blood or body fluids is anticipated chapter 10

Wearing gloves when in contact with blood or body fluids is anticipated

The nurse caring for a client who was just told she is 4 months pregnant and was unaware of the pregnancy. The nurse is reviewing any medications that the client may have taken in the past 5 months. Which characteristic of the medication(S) will the nurse assign the most significance when considering if the medication may be teratogenic? - How long the client took the drug prior to becoming a pregnant - Whether the medication is lipid or water-soluble - The route of the medication (oral, transdermal, or parenteral) - If the drug was a prescribed or taken without prescription chapter 5

Whether the medication is lipid or water-soluble

A client with a large decubitus ulcer asks the nurse, "How it is possible for a wound this deep to ever fully heal?" What is the nurse's best response? - Deep wounds like this require surgical intervention and skin grafting before they will be healed - With adequate resources, the body is capable of regenerating tissue and blood flow to the area over time - Once the first layer of new granulation tissue is formed, the speed of the repair will increase greatly - It will be difficult, but you have many experts planning your care so the chances of healing are good chapter 9

With adequate resources, the body is capable of regenerating tissue and blood flow to the area over time

A family of five vacationed together, but only the client contracted traveler's diarrhea. The client asks the nurse, "We all ate and drank the same things. Why am I the only one with this condition?" What should the nurse offer to the client as the most likely reason for susceptibility for this condition? - You are taking a proton pump inhibitor for gastric reflux disease - You are the only obese member of the family - You are taking a beta blocker for hypertension - You are male and your family members are all female chapter 10

You are taking a proton pump inhibitor for gastric reflux disease

A 70-year-old woman has received a diagnosis of chronic myelogenous leukemia (CML) after a clinical investigation sparked by the presence of leukocytosis in her routine blood work. What clinical course should the care provider tell her to expect? - You could remain the chronic stage of CML for several years before it accelerates and culminates in a crisis - You can expect your blood results, fatigue, and susceptibility to infection to gradually worsen over a few years - It's likely that this will give you chronic fatigue and malaise for the rest of your life, but that will probably be the extent of your symptoms - Unfortunately, your leukemia will likely enter a crisis mode within a few weeks if we don't treat it immediately chapter 24

You could remain the chronic stage of CML for several years before it accelerates and culminates in a crisis

A pregnant client learns she has fragile X syndrome through prenatal screening and asks how this might affect the unborn child. What information should the nurse include in the response. Select all that apply. - If you have a daughter, there is a 100% chance she will have the condition - If your husband is not a carrier, your children will not have fragile X syndrome - You have a 50% chance of passing the condition to a child of either sex - Males are less likely to be born with fragile X syndrome - Daughters are less severely affected than sons by fragile X syndrome chapter 5

You have a 50% chance of passing the condition to a child of either sex - Males are less likely to be born with fragile X syndrome - Daughters are less severely affected than sons by fragile X syndrome

A client who is recovering from burn injuries is discussing his prognosis with a physician. Which teaching point about expectations for healing should the physician offer? - You'll find that your new tissue is more elastic and fragile than the rest of your skin - You may find that the scar is a bit smaller than the area of the wound - Once your healing is complete, your skin will be just as strong as before your accident - The final remodeling phase of healing may last up to 3 months in your case chapter 9

You may find that the scar is a bit smaller than the area of the wound

A health care professional has recommended biofeedback to a client as a method of dealing with the high levels of stress. Which explanation should the nurse offer the client to certain biofeedback treatment? - You will become aware of the increased skin temperature that accompanies anxiety - You will be taught how to gain control over skeletal muscle contractions - You might be asked to use electrocardiogram as part of the therapy - The goal is to reduce the amount of daily stress to which you are exposed chapter 7

You will be taught how to gain control over skeletal muscle contractions

As part of her prenatal care, a pregnant woman and her partner are being taught by a community health nurse. Which of the following points about the teratogenic effects of different substances should the nurse include in his teaching? - Your developing baby is most vulnerable during the first 2 months of your pregnancy. - You need to be very careful with vitamin D and its derivatives - Keep in mind that a high percentage of genetic abnormalities are attributable to drug origins - Your best option is to avoid using any drugs during your pregnancy chapter 5

Your developing baby is most vulnerable during the first 2 months of your pregnancy.

A client has been diagnosed with rheumatoid arthritis and asks the nurse what causes this condition. What is the nurse's best response? - Your white blood cells are failing to respond to the increased inflammation in your joints - Your immune system is attacking your own tissues as if they were harmful organisms - Your joint tissues are releasing foreign chemicals, triggering your immune system to attack the joint - This is a condition where chronic inflammation causes the loss of bone and joint tissues chapter 11

Your immune system is attacking your own tissues as if they were harmful organisms

The nurse is caring for a client who has been living with Parkinson disease for the past 10 years and is being treated with levodopa-carbidopa. The client often leaves the unit with family for extended periods. What should the nurse prioritize when teaching the client? - Frequent assessments are needed to prevent complications related to your condition - It is essential that you eat a healthy diet with adequate amounts of protein if dining out - Due to your risk for falls, it is recommended you do not leave the unit with family - Your medication needs to be taken at equal intervals to reduce symptom fluctuations chapter 15

Your medication needs to be taken at equal intervals to reduce symptom fluctuations

A nurse on a cardiac unit is has noted that client's potassium level is 6.1 mEq/L (6.1 mmol/L). The nurse has notified the physician and removed the banana from the client's meal tray. When explaining and the nursing actions to the client, which statement is appropriate? - The amount of potassium in your blood is too high, but I will change your intravenous fluids - Your potassium level is high so I need you to tell me if you feel numbness, tingling, or weakness - I need to monitor you for signs of high potassium, tell me if you ever feel as if your heart is beating quickly - Your potassium levels in your blood are high than they should be, which brings a risk of changes in kidney function chapter8

Your potassium level is high so I need you to tell me if you feel numbness, tingling, or weakness

A 20 year-old has been diagnosed with astrocytic brain tumor located in the brainstem. Which statement by the oncologist treating the client is most accurate? - This is likely a result of a combination of heredity and lifestyle - Our treatment plan will depend on whether your tumor is malignant or benign - The major risk that you face is metastases to your lungs, liver, or bones - Your prognosis will depend on whether we can surgically resect your tumor chapter 16

Your prognosis will depend on whether we can surgically resect your tumor

A baseball player was hit in the head with a bat during practice. In the emergency department, the physician tells the family that he has a "coup" injury. How will the nurse explain this to the family so they can understand? - It's like squeezing an orange so tight that the juice runs out of the top - When the bat hit his head, his neck jerked backward causing injury to the spine - Your son has a huge laceration inside his brain where the bat hit his skull - Your son has a contusion of the brain at the site where the bat hit his head chapter 16

Your son has a contusion of the brain at the site where the bat hit his head

The parents of a 3-year-old boy have brought him to a pediatrician for assessment of the boy's late ambulation and frequent falls. Subsequent muscle biopsy has confirmed a diagnosis of Duchenne muscular atrophy. Which teaching point should the physician include when explaining the child's diagnosis to his parents? - Your son will be prone to heart problems and decreased lung function because of this - His muscles will weaken and will visibly decrease in size relative to his body size throughout his childhood - Your son's muscular dystrophy is a result of faulty connections between muscles and the nerves that normally control them - He'll require intensive physical therapy as he grows up and there's a good chance that he will outgrow this problem as he develops chapter 15

Your son will be prone to heart problems and decreased lung function because of this

A surgeon is explaining to the parents of a 6-year-old boy the rationale for the suggestion of removing the boy's spleen. Which teaching point would be most accurate? - Your son's spleen is inappropriately destroying the platelets from his blood and needs to be removed - We think that his spleen is inhibiting the production of platelets by his bone marrow - We believe that your son's spleen is causing the destruction of many of his blood platelets, putting him at a bleeding risk - Your son's spleen is holding on to too many of his platelets so they're not available for clotting chapter 22

Your son's spleen is holding on to too many of his platelets so they're not available for clotting

Laboratory testing is ordered for a male patient during a clinic visit for routine follow-up assessment of hypertension. When interpreting lab values, the nurse knows: - if the lab results is above the 50% distribution, the result is considered elevated - a normal value represents the test results that fall within the bell curve - all lab values are adjusted for gender and weight - if the result of a very sensitive test is negative, that does not mean the person is disease free chapter 1

a normal value represents the test results that fall within the bell curve

A clinician who works on a cardiac care unit of a hospital is providing care for a number of clients. Which client most likely has a genetic disorder arising from inheritance of a single gene? - a short, thin, 56 year old woman with hypertension - an overweight, middle aged male smoker with coronary artery disease - a tall, thin, myopic 28 year old woman with mitral valve prolapse - a thin, middle-aged nonsmoking man with a repaired atrial septal defect chapter 5

a tall, thin, myopic 28 year old woman with mitral valve prolapse

The health care provider is discussing the treatment protocol using ionizing radiation to treat the client's cancer. The client asks, "What side effect can occur with this treatment?" Which response is accurate? Select all that apply. - excessive diarrhea - new onset seizures - kidney stone formation - photosensitivity - low blood counts chapter 3

excessive diarrhea and low blood counts

A nurse is triaging clients at a disaster site. Local facilities have different specialized units. To what facility should the nurse send a client who has sustained an electrical injury to his left thigh? - Neurology unit - Cardiac care unit - Burn unit - Surgical intensive care unit chapter 3

burn unit

The nurse is planning for a 6-hour-old neonate who has been born with cleft palate. What aspect of care should the nurse prioritize? - infection control - positive body image - adequate nutrition - managed parental anxiety chapter 5

adequate nutrition

A home health nurse is making a visit to a family with an 8-month-old infant with severe motor deterioration. The physician has diagnosed the infant with Tay-Sachs disease. The parents are asking the nurse why this happened. The nurse will base her answers knowing the root cause of Tay-Sachs is: - high exposure to lead in the home environment - an increase in bilirubin retention leading to destruction of RBCs - an enzyme defect causing abnormal lipid accumulation in the brain - hypoxia caused by placing the infant on his or her abdomen during sleep chapter 3

an enzyme defect causing abnormal lipid accumulation in the brain

A nurse is teaching a group of older adults about the value including foods containing antioxidants in their diet. which statement best captures the rationale underlying the nurse's advice? - antioxidants prevent the formation of superoxide dismutase - antioxidants react nonspecifically with molecules - antioxidants prevent the occurrent of cell dysplasia - antioxidants inhibit the actions of reactive oxygen species chapter 3

antioxidants inhibit the actions of reactive oxygen species

The nurse is providing care for a client with a diagnosis of amyotrophic lateral sclerosis (ALS). The nurse recognizes that which mechanism is suspected to play a role in the cellular death associated with ALS? - liquefaction necrosis - causeous necrosis - apoptosis - hypoxic cell injury

apoptosis

A woman gives birth to a small infant with a malformed skull, The infant grows abnormally slowly and shows signs of substantial cognitive and intellectual deficits. The child has also facial abnormalities that become more striking as it develops. What might you expect to find in the mother's pregnancy history? - actives herpes simplex infection - chronic cocaine use - folic acid deficiency - chronic alcohol use chapter 5

chronic alcohol use

A nurse is considering setting up a screening program for a specific health condition in a population. What characteristic of the condition would need to be true for the nurse to justify screening a population? The condition should: - be curable with available treatment - have a high mortality rate - have a noninvasive diagnostic test available - be asymptomatic at an early stage chapter 1

be asymptomatic at an early stage

A 7-year-old boy that is admitted to the hospital with a suspected diagnosis of lead toxicity. Which assessment finding is most congruent with the client's diagnosis? - diffuse muscle pain - decreased deep tendon reflexes - white blood cells (wbc) - hemoglobin 9.9 g/dL (99 g/L) chapter 3

hemoglobin 9.9 g/dL (99 g/L)

The public health nurse is designing a course about risk factors for various chronic illnesses. For risk factors about which chronic illness will the nurse consult the Framingham study? - breast cancer - type 2 diabetes mellitus - cardiovascular disease - chronic obstructive pulmonary disease (COPD)

cardiovascular disease

An epidemiologist is conducting a program of research aimed at identifying factors associated with incidence and prevalence of congenital cardiac defects in infants. The researcher has recruited a large number of mothers whose infants were born with cardiac defects as well as mothers whose infants were born with cardiac defects as well as mothers whose infants were born with healthy hearts. The researcher is comparing the nutritional habits of all the mothers while their babies were in utero. Which type of study is the epidemiologist most likely conducting? - risk factor study - case-control study - cohort study - cross-sectional study chapter1

case-control study

A nurse practitioner is working in a crowded neighborhood where the population is primarily immigrants from China. The nurse has designed a research study to follow children from kindergarten to the age of 25. She is going to be looking at their diet, successful progression in school, health practices, and development of disease, to name a few items. This type of research is known as: - case-control study - epidemiologic study - cross-sectional study - cohort study chapter 1

cohort study

The nurse knows high incidences of infectious illnesses among the older adults who reside in a long term care facility are most likely to have diminished immune capacity because of: - decreased numbers and responsiveness of T lymphocytes - decreased antigen recognition by B lymphocytes - over-expression of cytokines and receptors - altered function in peripheral lymphocytes chapter 11

decreased numbers and responsiveness of T lymphocytes

A client diagnosed with a low-risk chronic lymphocytic leukemia (CLL) has recently developed thrombocytopenia. One of the medications utilized to treat this would be: - dexamethasone, a corticosteroid - cistaplatin, a chemotherapeutic - vincristine, a vinca alkaloid - doxorubicin, a cytoxic antibiotic chapter 24

dexamethasone, a corticosteroid

The spirochete Leptospira is primarily transmitted to farmers by: - direct contact with infected animals - exposure to spores in the environment - a mosquito bite - an airborne mechanism chapter 10

direct contact with infected animals

The nurse is caring for a client with Marfan syndrome. Which assessment finding should the nurse prioritize as requiring the most emergent intervention? A sudden: - complete loss of vision in a single eye - onset of confusion and altered level of consciousness - drops in heart rate from 80 to 60 beats per minute - drop in blood pressure from 130/90 to 95/50 mm Hg chapter 5

drop in blood pressure from 130/90 to 95/50 mm Hg

A nurse researcher is interested in the natural history of a disease being studied. What should the nurse focus on to best understand the condition's natural history? - following client cases from initial diagnosis and throughput treatment - examining outcomes across a wide variety of treatment approaches - interview clients for their first-hand experience with the condition - focusing on clients who did not receive treatment for the condition chapter 1

focusing on clients who did not receive treatment for the condition

Which client's signs and symptoms would allow a clinician to be most justified in ruling out stroke as a cause? An adult: - has had a gradual onset of weakness, headache, and visual disturbances over the last 2 days - states that her left arm and leg are numb and gait is consequently unsteady - experienced a sudden loss of balance and slurred speech - has vomited and complained of a severe headache chapter 16

has had a gradual onset of weakness, headache, and visual disturbances over the last 2 days

The nurse knows which statement best describes a characteristic trait of Rickettsiae related to Rocky Mountain spotted fever? Rickettsiae: - have a distinct spiral-shaped morphology - are eukaryotic - have both RNA and DNA - are neither gram-negative nor gram-positive chapter 10

have both RNA and DNA

A 10 year old child with strep throat asks the nurse, "Why are there large bumps (lymph nodes) on my neck when my throat gets sore?" The nurse replies that lymph nodes: - helps your tonsils get bigger with cells that will bring immune cells into your throat to prevent any other infections - bring in cells into the lymph node (your bump) to stop the germs from going anywhere else in the body - bring all kind of good cells to your throat so that they can wall the strep off and keep the germs from getting any food or water - help your body fight off infections by allowing special cells (lymphocytes and macrophages) to move through the lymph chain and engulf and destroy germs chapter 11

help your body fight off infections by allowing special cells (lymphocytes and macrophages) to move through the lymph chain and engulf and destroy germs

A new older female client at a long term care facility has a diagnosis of neurofibromatosis type 1. As part of the intake assessment protocol for the facility, the clinical educator is teaching the care staff about the diagnosis. Which statement most accurately conveys an aspect of neurofibromatosis? - the client is likely to be photosensitive as a result of the disease - she is living with an example of an autosomal recessive disorder - the neurofibroma lesions are unsightly for the client, but they are not painful - her diagnosis puts her at higher risk of developing a malignant neoplasm chapter 5

her diagnosis puts her at higher risk of developing a malignant neoplasm

A particular disease has a debilitating effect on the ability of sufferers to perform their activities of daily living, and is a significant cause of decreased quality of life. However, few people die as a result of the disease's direct effects. There are hundreds of thousands of Americans living with the disease but relatively few new cases in recent years. Which statement best conveys an accurate epidemiologic characterization of the disease? - low mortality, high morbidity, high incidence, low prevalence - high morbidity, low mortality, high prevalence, low incidence - high mortality, low morbidity, high incidence, low prevalence - low mortality, high morbidity, low prevalence, high incidence chapter 1

high morbidity, low mortality, high prevalence, low incidence

The laboratory technologists are a discussing a new blood test that helps establish a differential diagnosis between shortness of breath with a cardiac etiology and shortness of breath with a respiratory/pulmonary etiology. A positive result is purported to indicate a cardiac etiology. The marketers of the test report that 99.85 of clients who have confirmed cardiac etiologists test positive in the test. however 1.3% of clients who do not have cardiac etiologies for their shortness of breath also test positive. which statement best characterizes this blood test. - low validity, high reliability - high sensitivity, low specificity - high specificity, low reliability - high sensitivity, low reliability chapter 1

high sensitivity, low specificity

The nurse is teaching new nursing assistants on the unit about the phenomenon of muscle of hypertrophy. Which client on the unit is most likely to experience muscle hypertrophy? A client with: - possible rejection symptoms following a liver transplant - peripheral edema secondary to heart failure (HF) - hypertension, obesity, and decreased activity tolerance - urinary incontinence following a cerebrovascular accident (CVA) Chapter 3

hypertension, obesity, and decreased activity tolerance

A client has stage IV cancer but further testing is needed to determine the site of origin of this metastatic tumor., Which form of testing will the health care provider be discussing with this client? - Sentinal node recovery - Papanicolaou test - immunohistochemistry - microarray technology chapter 6

immunohistochemistry

Which statement most accurately conveys an aspect of cell injury due to impaired calcium homeostasis? - low calcium levels cause an activation of damaging enzymes - injured cells tend to accumulate calcium - normal intracellular calcium ion levels are higher than extracellular levels - ischemia and certain toxins cause a decrease in cytosolic calcium chapter 3

injured cells tend to accumulate calcium

The nurse is educating a client who is undergoing gamma knife radiosurgery for a brain metastasis. What advantage to this procedure should the nurse share when comparing it to other conventional treatments? - less explosive vomiting - lessen the number of migraine headaches per day - less cognitive dysfunction - less chance of developing focal seizures chapter 6

less cognitive dysfunction

A worker in a warehouse is trying to have children but think he or she has handled "mercury" while cleaning equipment. Which statement by the occupational nurse is most appropriate at this time? - infertility is caused by too much mercury exposure - most mercury toxicity involves central nervous system changes - you should have all the mercury fillings in your teeth changed to newer products - usually mercury toxicity appears as skin rashes and lesions chapter 3

most mercury toxicity involves central nervous system changes

The nurse knows which statement listed below relative to a client with malignant melanoma treated with alpha interferon (IFN-a) is accurate? Alpha interferon (IFN-a): - helps keep all the blood levels at a higher level - plays an important role in the modulation of the inflammatory response - controls the migration of leukocytes to their primary site - will kill certain microorganisms that may help spread the cancer chapter 11

plays an important role in the modulation of the inflammatory response

As part of a community class, student nurses are developing curriculum to teach expectant parents the importance of having their child properly secured in a child safety seat. During the class, the students are going to have a safety officer examine the car seats that the parents have installed in their vehicle. This is an example of which type of prevention? - prognosis enhancement - primary prevention - secondary prevention - tertiary prevention chapter 1

primary prevention

When looking at a granulocyte under a microscope, the anatomy student would describe it as: - having no nuclei - lacking granules - shaped like a sphere with a multilobar nuclei - having a kidney-shaped nucleus chapter 21

shaped like a sphere with a multilobar nuclei

A client who has a diagnosis of lung cancer is scheduled to begin radiation treatment. the nurse knows that which statement about potential risks of radiation is most accurate? - the changes that you might see are normally irreversible - the unwanted effects will be limited to exposed portions of your skin - sometimes you might find that your blood takes longer to clot than normal - some clients experience long-term irritation of skin adjacent to the treatment site chapter 3

some clients experience long-term irritation of skin adjacent to the treatment site

A health care provider has prescribed blood products for a trauma client with a history of selective immunoglobulin A deficiency (SIGAD) who is going into hypovolemic shock. Which blood product is most appropriate for the nurse to infuse? - platelets from a single donor - fresh frozen plasma from multiple donors - specially washed erythrocytes from normal donor - cryoprecipitate from pooled donors chapter 12

specially washed erythrocytes from normal donor

A multidisciplinary healthcare team operates a program aimed at the prevention, identification, and treatment of diabetes on a large Indian reservation. Which aspect of the program would most likely be classified as secondary prevention? - administering oral antihyperglycemic medications to clients who have a diagnosis of diabetes - staffing a booth where community residents who are attending a baseball tournament can have their blood glucose levels checked - teaching school children how a nutritious, traditional diet can lessen their chances of developing adult-onset diabetes - regularly scheduled wound dressing changes for clients who have foot ulcers secondary to peripheral neuropathy and impaired wound healing chapter 1

staffing a booth where community residents who are attending a baseball tournament can have their blood glucose levels checked

Which enzyme listed below is responsible for the ability of cancer cells to resist aging and contributes to cellular immortality that is so characteristic of this disease process? - hydrolase - isomerase - telomerase - oxidoreductase chapter 3

telomerase

An occupational therapist conducts a group therapy program called Mindworks with older adults who have diagnosis of dementia and Alzheimer's disease. The goal of the group is to slow the cognitive decline of clients by engaging them in regular, organized mental activity such as reading maps and solving puzzles. How would the program most likely be characterized? - primary prevention - prognosis prevention - secondary prevention - tertiary prevention chapter 1

tertiary prevention

A 6-year-old girl with a diagnosis of Marfan syndrome is being assessed at a community health clinic. Which assessment would be the health care professional's lowest priority? - a test of the child's visual acuity - tests of kidney function - cardiovascular assessment - a musculoskeletal assessment chapter 5

tests of kidney function

A 68-year-old male client with aortic stenosis secondary to calcification of the aortic valve is receiving care. Which statement best captures an aspect of this client's condition? - increased calcium intake over time may have contributed to the problem - paget disease, cancer with metastases, or excess vitamin D may have contributors - the client has possibly exhibited phophate retention leading to calcium deposits - the client has possibly undergone damage as a result of calcification following cellular injury chapter 3

the client has possibly undergone damage as a result of calcification following cellular injury

A male international business traveler has returned from a trip to Indonesia. While there, he hired a prostitute for companionship and engages in unprotected sex on more than one occasion. unbeknownst to him, this prostitute harbored the hepatitis C virus. Upon return to the U.S., he exhibited no symptoms and returned to his usual activities. during this period of no outward symptoms, the disease would be classified as being in: - remission and unlikely to develop hepatitis C - the chronic phase of hepatitis C - the preclinical stage of disease - the clinical disease stage of hepatitis C chapter 1

the preclinical stage of disease

The nurse is teaching a group of new mothers about postpartum nutrition, when one of the clients states that she was told to avoid eating fish too often due to the risk of mercury poisoning. Which response by the nurse most accurately addresses the clients concerns? - the risk of mercury toxicity from eating fish has been shown to be insignificant - you're right, it's best to avoid eating fish, especially when you are breast-feeding - provided you avoid salmon, you likely won't be putting yourself or your child at risk - there are some modest risks, but they are only associated with some long-living fish like tuna chapter 3

there are some modest risks, but they are only associated with some long-living fish like tuna

As part of an orientation to a genetic counseling practice, a group of medical students are differentiating between autosomal recessive disorders and autosomal dominant disorders. Which of the following statements is true of autosomal recessive disorders? - they can manifest when present in one or both gene pairs - there is a 1 in a 2 chance of an affected child in each pregnancy with an affected mother - they tend to have a more uniform symptomology than autosomal dominant disorders - the associated disorders are usually attributable to abnormalities in structural proteins chapter 5

they tend to have a more uniform symptomology than autosomal dominant disorders

The nurse knows a drug in a category identified as a colony-stimulating factor (CSF) helps: - produce cells that will be the first responder to protect against cancer formation - stimulate the person's immune system so that they can kill their own cancer cells - to stimulate bone marrow to produce large numbers of mature cells such as platelets and erythrocytes - cells engulf and digest microbes that want to attach to cell membranes and destroy normal cell function - chapter 11

to stimulate bone marrow to produce large numbers of mature cells such as platelets and erythrocytes

During a myocardial infarction (MI), a client with.a 97% occlusion of the left descending artery develops ventricular dysrhythmias due to the amount of ischemia occurring in the myocardium. While providing education about mi's, which statement is most accurate to share with this client? - The body will grow new genes thru the process of angiogenesis, thereby avoiding any permanent damage to the myocardium - once the oxygen supply has been occluded, cellular changes are irreversible even if oxygenation is restored - treatment needs to be sought immediately so the buildup of lactic acid is limited and cellular changes can be reversed - permanent damage will occur in the myocardium if the vessel is not opened within a 1-2 minute window following the occlusion chapter 3

treatment needs to be sought immediately so the buildup of lactic acid is limited and cellular changes can be reversed

Which individual is likely to have the best prognosis for recovery from his or her insult to the peripheral nervous system. An adult: - who suffered a bone-depth laceration to the shoulder during a knife attack - client who had nerves transected during surgery to remove a tumor from the mandible - client who developed rhabdomyolysis and ischemic injury after a tourniquet application - who had his forearm partially crushed by gears during an industrial accident chapter 15

who had his forearm partially crushed by gears during an industrial accident


Related study sets

Deca Business and Administration 2012

View Set

Pseudocode Algorithm Workbench Ch 6-11

View Set

Midterm-Management Information System

View Set

Prescribing for Pediatric Patients

View Set